49
Discussion and References Question #1 ANSWER=A ______________________________________________________________________________ Gross hematuria in a newborn is classically due to either renal vein thrombosis, renal artery thrombosis, acute cortical necrosis, or birth trauma to a pre-existing hydronephrotic kidney. The patient's history is greatly helpful in determining the cause of the hematuria. Renal vein thrombosis will usually develop in large birth weight infants born from mothers with gestational diabetes or as a consequence of neonatal gastroenteritis. Loss of intravascular volume due to obligatory water loss from an osmotic diuretic (maternal induced glucosuria of the neonate) or infantile diarrhea resulting in hemoconcentration can result in the development of renal vein thrombosis. The thrombosis will start in the medulla of the kidney and is propagated proximally. The clot is not amenable to surgical removal. Renal artery thrombosis usually occurs secondary to blood clot developing on a indwelling umbilical artery catheter. Acute cortical necrosis is associated with neonatal shock typically found after a maternal catastrophe, e.g., abruption placentae with maternal and fetal shock, etc. In this patient, the ultrasound findings of an enlarged right kidney without hydronephrosis rules out the possibility of a trauma due to a pre-existing hydronephrotic kidney. The findings of no venous flow on renal ultrasound and non-visualization of the ipsilateral kidney on renal scan are classic for renal vein thrombosis. Renal venous thrombosis is frequently associated with hypertension, consumptive coagulopathy and hypovolemia. Prompt I.V. rehydration and I.V. antibiotic prophylaxis to help prevent bacterial seeding of the necrotic kidney are recommended as the initial steps in patient management. The platelet count will return to normal when the clotting process ceases. The use of intravascular thrombolytic agents or heparin in a neonate is associated with the complication of neonatal hemorrhage and are used only in dire circumstances with bilateral renal involvement. Due to the profound hypertension frequently coexisting with RVT, use of these agents are fraught with the complication of neonatal intracranial hemorrhage and death. Nephrectomy is indicated if hypertension cannot be medically controlled and the contralateral kidney is normal. Lee RS, Borer JG: Perinatal urology, Wein, AJ, Kavoussi LR, Novick AC, Partin AW, Peters CA (eds): CAMPBELL-WALSH UROLOGY, ed 10. Philadelphia, Elsevier Saunders, 2012, vol 4, chap 114, pp 3060-3061. Question #2 ANSWER=A ______________________________________________________________________________ In epispadias, the neurovascular bundles are dorsally and laterally displaced on the proximal corpora and laterally displaced on the distal corpora. The nerves are readily identified during degloving of the penis at the time of epispadias repair. To correct the dorsal curvature that is routinely found in epispadias, the distal corporal bodies are medially rotated. The medial rotation of the corpora bodies during the repair moves the neurovascular bundles towards the more normal dorsal mid-line location. Knowing the location of the penile nerve supply in children with epispadias is key to preserving erectile function in children with this birth defect.

Live Course Handout.cfm Second Part

Embed Size (px)

DESCRIPTION

urology pediatric mcq part 2

Citation preview

Page 1: Live Course Handout.cfm Second Part

Discussion and References Question #1 ANSWER=A ______________________________________________________________________________ Gross hematuria in a newborn is classically due to either renal vein thrombosis, renal artery thrombosis, acute cortical necrosis, or birth trauma to a pre-existing hydronephrotic kidney. The patient's history is greatly helpful in determining the cause of the hematuria. Renal vein thrombosis will usually develop in large birth weight infants born from mothers with gestational diabetes or as a consequence of neonatal gastroenteritis. Loss of intravascular volume due to obligatory water loss from an osmotic diuretic (maternal induced glucosuria of the neonate) or infantile diarrhea resulting in hemoconcentration can result in the development of renal vein thrombosis. The thrombosis will start in the medulla of the kidney and is propagated proximally. The clot is not amenable to surgical removal. Renal artery thrombosis usually occurs secondary to blood clot developing on a indwelling umbilical artery catheter. Acute cortical necrosis is associated with neonatal shock typically found after a maternal catastrophe, e.g., abruption placentae with maternal and fetal shock, etc. In this patient, the ultrasound findings of an enlarged right kidney without hydronephrosis rules out the possibility of a trauma due to a pre-existing hydronephrotic kidney. The findings of no venous flow on renal ultrasound and non-visualization of the ipsilateral kidney on renal scan are classic for renal vein thrombosis. Renal venous thrombosis is frequently associated with hypertension, consumptive coagulopathy and hypovolemia. Prompt I.V. rehydration and I.V. antibiotic prophylaxis to help prevent bacterial seeding of the necrotic kidney are recommended as the initial steps in patient management. The platelet count will return to normal when the clotting process ceases. The use of intravascular thrombolytic agents or heparin in a neonate is associated with the complication of neonatal hemorrhage and are used only in dire circumstances with bilateral renal involvement. Due to the profound hypertension frequently coexisting with RVT, use of these agents are fraught with the complication of neonatal intracranial hemorrhage and death. Nephrectomy is indicated if hypertension cannot be medically controlled and the contralateral kidney is normal. Lee RS, Borer JG: Perinatal urology, Wein, AJ, Kavoussi LR, Novick AC, Partin AW, Peters CA (eds): CAMPBELL-WALSH UROLOGY, ed 10. Philadelphia, Elsevier Saunders, 2012, vol 4, chap 114, pp 3060-3061. Question #2 ANSWER=A ______________________________________________________________________________ In epispadias, the neurovascular bundles are dorsally and laterally displaced on the proximal corpora and laterally displaced on the distal corpora. The nerves are readily identified during degloving of the penis at the time of epispadias repair. To correct the dorsal curvature that is routinely found in epispadias, the distal corporal bodies are medially rotated. The medial rotation of the corpora bodies during the repair moves the neurovascular bundles towards the more normal dorsal mid-line location. Knowing the location of the penile nerve supply in children with epispadias is key to preserving erectile function in children with this birth defect.

Page 2: Live Course Handout.cfm Second Part

Gearhart JP, Mathews RI: Exstrophy-epispadias complex, Wein, AJ, Kavoussi LR, Novick AC, Partin AW, Peters CA (eds): CAMPBELL-WALSH UROLOGY, ed 10. Philadelphia, Elsevier Saunders, 2012, vol 4, chap 124, pp 3349-3352. Question #3 ANSWER=E ______________________________________________________________________________ Stones are common in bowel augmentations and neobladders in children, and most are infection-related stones forming around a mucous core nidus. Metabolic evaluations performed in patients with multiple episodes of recurrent vesicolithiasis post neobladder formation will frequently find co-existing hypocitraturia in up to 30%. Treatment of hypocitraturia in patients with recurrent bladder stones has not been found to be beneficial unless there is concurrent evidence of upper tract nephrolithiasis. Treatment of bladder calculi after removal is best treated by optimizing the catheterization schedule and daily bladder irrigations to decrease mucus formation that serves as a nidus for stone formation. Palmer LS, Franco I, et al: Urolithiasis in children following augmentation cystoplasty. J UROL, 1993;150:726.Adams MC, Joseph DB: Urinary tract reconstruction in children, Wein, AJ, Kavoussi LR, Novick AC, Partin AW, Peters CA (eds): CAMPBELL-WALSH UROLOGY, ed 10. Philadelphia, Elsevier Saunders, 2012, vol 4, chap 129, pp 3483-3484. Question #4 ANSWER=D ______________________________________________________________________________ A 1 cm stricture is nearly always suitable for excision and direct anastomosis, and this technique provides the best long-term results with a success of >90%. Success of DVIU is highly variable and dependent upon multiple factors: location (highest success in bulbar urethra), length of the stricture <1 cm, diameter of stricture (>12 F), and density of the stricture. Overall success of a DVIU as defined by lack of symptoms and normal flow rates is reported as <35% at two years. Studies comparing urethra dilation to a DVIU in the management of short, wide caliber strictures show comparable long-term results with neither procedure having a definitive advantage. Steroid injection delays stricture recurrence; however, long-term follow-up reveals similar failure rates, showing steroids only delaying the time to failure. Vascularized flap urethroplasty, or grafts with buccal mucosa may be appropriate for longer strictures, but is most likely not needed in this patient. Husmann DA: Pediatric genitourinary trauma, Wein, AJ, Kavoussi LR, Novick AC, Partin AW, Peters CA (eds): CAMPBELL-WALSH UROLOGY, ed 10. Philadelphia, Elsevier Saunders, 2012, vol 4, chap 138, pp 3749-3750. Question #5 ANSWER=B ______________________________________________________________________________ This boy has secondary rather than primary VUR. Therapy should be directed toward improving bladder compliance rather than correcting the VUR. Correction of bladder dynamics should take precedent over

Page 3: Live Course Handout.cfm Second Part

consideration of VUR. Indeed, non-dilating VUR may spontaneously resolve following improvement of bladder compliance by appropriate pharmacotherapy. Although augmentation and or onabotulinumtoxinA will improve detrusor compliance and may well be necessary, the first step should be to maximize nonoperative pharmacotherapy and then re-evaluate with repeat urodynamics. The combination of imipramine and oxybutynin will improve compliance over the use of either medication alone; however, combined pharmacotherapy should not be pursued until a single agent is brought to maximal dosage tolerance. MacLellan DL, Bauer SB: Neuropathic dysfunction of the lower urinary tract, Wein, AJ, Kavoussi LR, Novick AC, Partin AW, Peters CA (eds): CAMPBELL-WALSH UROLOGY, ed 10. Philadelphia, Elsevier Saunders, 2012, vol 4, chap 128, pp 3437-3440. Question #6 ANSWER=D ______________________________________________________________________________ The most common presentation of congenital adrenal hyperplasia in boys is isosexual precocious puberty. The infant will appear normal at birth, but by three years of age, he will develop both sexual and somatic precocity. This is manifest by enlargement of the genitalia, sparing the testes, with development of pubic hair and a deepening voice. This is all due to androgen excess. The basic defect is an inability to synthesize corticosteroids; therefore, serum cortisol is low. A pituitary tumor, Leydig cell tumor, steroid abuse, and adrenocortical carcinoma all might cause precocious puberty, but would not result in a low serum cortisol level. Diamond DA, Yu RN: Sexual differentiation: Normal and abnormal, Wein, AJ, Kavoussi LR, Novick AC, Partin AW, Peters CA (eds): CAMPBELL-WALSH UROLOGY, ed 10. Philadelphia, Elsevier Saunders, 2012, vol 4, chap 133, p 3616. Question #7 ANSWER=E ______________________________________________________________________________ This teenager has progressive hydronephrosis despite an augmentation cystoplasty. The differential diagnosis includes insufficient catheterization due to noncompliance with medical directives, UVJ obstruction, VUR, uncontrolled detrusor instability, and/or poor detrusor compliance. It is noteworthy that evaluations comparing ileal, ascending colon and sigmoid bladder augmentations reveal that sigmoid augmentations are the most likely to develop complications of detrusor instability on long-term follow-up. This increased detrusor overactivity may be associated with the onset of urinary incontinence and/or upper tract deterioration. Spinal cord tethering is unlikely to be the cause of the clinical findings, particularly after bladder augmentation. Although a catheterization diary, diuretic renogram, and VCUG would all be helpful, the most direct way of making the diagnosis is with videourodynamics. The videourodynamic results combined with the findings from a catheterization diary will be extremely useful in the clinical management of this patient.

Page 4: Live Course Handout.cfm Second Part

MacLellan DL, Bauer SB: Neuropathic dysfunction of the lower urinary tract, Wein, AJ, Kavoussi LR, Novick AC, Partin AW, Peters CA (eds): CAMPBELL-WALSH UROLOGY, ed 10. Philadelphia, Elsevier Saunders, 2012, vol 4, chap 128, pp 3433-3436. Question #8 ANSWER=A ______________________________________________________________________________ The skin of the prepuce and penile shaft and their accompanying dartos fascia is highly mobile because of their loose attachment to Buck's fascia. The preputial and penile skin blood supply is derived from the external pudendal artery arising from the femoral vessels. These vessels enter the base of the penis to run longitudinally in the dartos fascia as a richly anastomotic network. Thus, penile skin and or its associated dartos fascia may be mobilized on a vascular pedicle as the ideal tissue to aid in urethral reconstruction. The internal pudendal artery arises from the internal iliac (hypogastric) artery and gives rise to the common penile artery. The common penile artery subdivides into three branches to supply the erectile bodies. The bulbourethral artery penetrates the perineal membrane to enter the proximal corpora spongiosum, it supplies the urethra, corpora spongiosum, and glans. The cavernosal artery (deep penile artery) pierces the corporal body in the penile hilum, will run near the center of corpus cavernosum, and is the chief blood supply to the cavernous sinuses. It does give off collateral branches that will feed into the circumferential branches that arise from the dorsal artery of the penis and, therefore, aids in the blood supply to the corpus spongiosa. The dorsal artery of the penis passes between the crus penis and the pubis to reach the dorsal surface of the corporal bodies. It runs between the dorsal vein and the dorsal penile nerve, and then attaches with them to the underside of Buck fascia. As it courses to the glans, it gives off cavernous branches and circumferential branches to the spongiosum and urethra. The rich blood supply of the triple vessels to the spongiosum allows safe division of the urethra during an excision and primary anastomotic urethral stricture repair. The inferior epigastric gives rise to the deep circumflex iliac, pubic, and cremasteric arteries. The first two supply the inguinal ligament and surrounding structures while the cremasteric artery supplies the cremasteric muscle, surrounding scrotum and gives accessory blood supply to the testicle. Chung BI, Sommer G, Brooks JD: Anatomy of the lower urinary tract and male genitalia, Wein, AJ, Kavoussi LR, Novick AC, Partin AW, Peters CA (eds): CAMPBELL-WALSH UROLOGY, ed 10. Philadelphia, Elsevier Saunders, 2012, vol 1, chap 2, p 65. Question #9 ANSWER=E ______________________________________________________________________________ This patient has Kallmann syndrome, clinically manifested by anosmia, diminished FSH and LH secretion, cryptorchidism, and micropenis. Bromocriptine is used to treat low LH-FSH secondary to pituitary tumors with excess prolactin secretion, and is not indicated in this scenario. Although hCG and GnRH would result in virilization, patients with Kallmann syndrome require daily injections, which are prohibitively expensive. The most cost-effective treatment modality to encourage penile growth and virilization is transdermal testosterone. If and when fertility is desired, testosterone may be discontinued and daily injections of hCG can be used as an alternative. hCG in this circumstance is used to increase

Page 5: Live Course Handout.cfm Second Part

sperm production. It is noteworthy that orchiopexy alone without concurrent gonadotropin stimulation or testosterone supplementation would not result in penile lengthening and virilization. Sabanegh E, Agarwal A: Male infertility, Wein, AJ, Kavoussi LR, Novick AC, Partin AW, Peters CA (eds): CAMPBELL-WALSH UROLOGY, ed 10. Philadelphia, Elsevier Saunders, 2012, vol 1, chap 21, p 638. Question #10 ANSWER=B ______________________________________________________________________________ SWL is considered one of the preferred treatment modalities for uncomplicated renal calculi less than or equal to 15 mm in the pediatric population. If used in the correct clinical scenario stone free rates in children exceed 80% and are higher than stone free rates for similar sized stones in adults. The superior success rates with SWL monotherapy in children compared with adults have been attributed to softer stone composition, smaller relative stone volume, increased ureteral compliance to accommodate stone fragments, small skin to stone distance, and smaller body volume to facilitate shock transmission. The ability for SWL to successfully treat lower pole renal calculi is highly dependent upon the pyelocalyceal angle of the lower pole calyx. If this angle exceeds 45 degrees, as in this patient, stone free rates of 80% can be expected. If the angle is more acute than 45 degrees, stone clearance rates decrease to <45%. Treatment of lower calyceal stones with an acute angle are problematic. Although ureteroscopic stone extraction can be considered it will almost invariably require pre-stenting of the ureter and be technically difficult to reach and is hypothesized to be at an increased risk for long term ureteral or urethral injuries especially in the prepubertal male. In patients with a lower pole acute pyelocalyceal angle of less than 45 degrees it is probably more efficacious to approach the stone via a direct percutaneous puncture. Partial nephrectomy is reserved for managing calyceal diverticuli that are inaccessible by either endoscopic or percutaneous approaches and associated with poorly functional segments. Ost MC, Schneck FX: Surgical management of pediatric stone disease, Wein, AJ, Kavoussi LR, Novick AC, Partin AW, Peters CA (eds): CAMPBELL-WALSH UROLOGY, ed 10. Philadelphia, Elsevier Saunders, 2012, vol 4, chap 135, pp 3669-3673.Donnella, SM, Harwood, LM, Webb, OR: Percutaneous management of calyceal diverticuli calculi: Technique and outcome. J ENDOUROL 1999;13:83-88.Sabnis, RB, Naik, R, Patel, SH, Sesai, MR, Bapat, SP: Extracorporeal shock wave lithotripsy for lower calyceal stones: Can clearance be predicted? BRI J UROL 1997;80:853-857.Clayman, R: Editorial Comments. Extracorporeal shock wave lithotripsy for stones: Can clearance be predicted? J UROL 1998;160:629. Question #11 ANSWER=A ______________________________________________________________________________ In a patient with known uric acid or cysteine stones (provided the kidney is functional and not obstructed), alkalization of the urine is the most appropriate initial step. Subsequent treatment is based upon the stone size, location, renal anatomy, stone composition and skin to stone distance. Relative indications for PCNL as the primary treatment modality include large upper tract stone burden (>1.5

Page 6: Live Course Handout.cfm Second Part

cm), a lower pole calculi >1 cm, concurrent anatomic abnormality that would impair urinary drainage, a skin to stone distance of >10 cm, and a stone composition of either cystine or struvite. Although there are no randomized trials directly comparing open pyelolithotomy versus PCNL in stones meeting the aforementioned criteria, serial case series reveal that percutaneous stone removal is associated with reduced convalescence, reduced narcotic requirements, and hospitalization. Open or laparoscopic pyelolithotomy should be reserved for cases where multiple procedures by less invasive procedures would be necessary, and on the rare occasions where endoscopic or percutaneous approaches are not reasonable due to anatomic variations. Ureteroscopy and laser lithotripsy can be performed, but may require additional anesthetics for both placement of a ureteral stent to passively dilate the ureter and/or to remove a stent placed at the time of ureteroscopy. Additionally, in a young prepubertal male, there is a small but finite risk (one to three percent) of both ureteral and urethral injury and subsequent stricture formation. The stone free rate after percutaneous stone removal of a large intrarenal stone (>1.5 cm) approaches 95%, significantly higher than management of similar sized stones by ureteroscopy, the latter approaching 70% for stone burdens >1.5 cm. Ost MC, Schneck FX: Surgical management of pediatric stone disease, Wein, AJ, Kavoussi LR, Novick AC, Partin AW, Peters CA (eds): CAMPBELL-WALSH UROLOGY, ed 10. Philadelphia, Elsevier Saunders, 2012, vol 4, chap 135, p 3676.Preminger GM, Tiselius H-G, Assimos DG, et al: 2007 Guideline for the management of ureteral calculi. AUA GUIDELINES. American Urological Association Education and Research, Inc, 2007. http://www.auanet.org/education/guidelines/ureteral-calculi.cfmPreminger GM, Assimos DG, Lingeman JE: AUA Guideline on the management of staghorn calculi: Diagnosis and treatment recommendations. AUA GUIDELINES. American Urological Association Education and Research, Inc, 2005.http://www.auanet.org/education/guidelines/staghorn-calculi.cfm Question #12 ANSWER=A ______________________________________________________________________________ Many nutritional problems may occur as the result of a loss of significant intestinal absorptive surface. These include Vitamin B12 deficiency (especially if the last 15-20 cm of the distal ileum is used), malabsorption of bile salts, and loss of Vitamins A, D, E and K. Vitamin K deficiency may result in an increased prothrombin time and clotting deficiency. It is therefore recommended that a prothrombin time be checked prior to any major surgical procedure in a patient with a history of neobladder or bladder augmentation. Use of heparin prolongs the partial thromboplastin time. Bleeding time assesses platelet function, and platelet count confirms normal platelet numbers. Platelet function is not altered by loss of the terminal ileum. Hemophilia A is caused by a deficiency of clotting factor VIII, which is also not affected by loss of the terminal ileum. Adams MC, Joseph DB: Urinary tract reconstruction in children, Wein, AJ, Kavoussi LR, Novick AC, Partin AW, Peters CA (eds): CAMPBELL-WALSH UROLOGY, ed 10. Philadelphia, Elsevier Saunders, 2012, vol 4, chap 129, pp 3480-3482. Question #13 ANSWER=E ______________________________________________________________________________

Page 7: Live Course Handout.cfm Second Part

A deep sacral dimple raises the concern of underlying spinal cord pathology. An ultrasound of the spine in the neonate is an accurate method of diagnosing a congenital spinal cord anomaly, as well as the least expensive. This test the preferred method to evaluate the spinal canal until the spinal column begins to ossify at three months of age. An abnormality such as a thickened or fatty filum is readily apparent on an ultrasound, and the lower extent of the spinal cord should be at or above L2- L3 interspace to rule-out a tethered cord. CT and MRI scans can establish the diagnosis as well, but will expose the child to radiation or require sedation, and they are more expensive. LS spine films do not delineate the status of the cord itself. Observation is not acceptable, as there is the potential for progressive deterioration of lower extremities and bowel and bladder function if the diagnosis of cord tethering is missed. MacLellan DL, Bauer SB: Neuropathic dysfunction of the lower urinary tract, Wein, AJ, Kavoussi LR, Novick AC, Partin AW, Peters CA (eds): CAMPBELL-WALSH UROLOGY, ed 10. Philadelphia, Elsevier Saunders, 2012, vol 4, chap 128, p 3447. Question #14 ANSWER=B ______________________________________________________________________________ This patient with hypospadias and progressive renal insufficiency has renal biopsy findings characteristic of Denys-Drash syndrome. These patients have a characteristic mutation of the WT1 gene. In patients that are dialysis dependent who develop a unilateral Wilms' tumor, a bilateral nephrectomy is recommended as management. Approximately 75% of normal contralateral kidneys are found to have widespread nephrogenic rests with the hypothetical potential for developing a metachronous Wilms' tumor. Given the predisposition to tumor formation, renal biopsy could upstage the disease, and is not indicated prior to nephrectomy. Potential for testicular tumors are also high within these patients. Specifically, these patients are at increased risk for a gonadoblastoma within their streak gonad during childhood and seminoma development within the postpubertal testicle. Tank E, Melvin T: The association of Wilms' tumor with nephrologic disease. J PED SUR 1990;25:724. Ritchey ML, Shamberger RC: Pediatric urologic oncology, Wein, AJ, Kavoussi LR, Novick AC, Partin AW, Peters CA (eds): CAMPBELL-WALSH UROLOGY, ed 10. Philadelphia, Elsevier Saunders, 2012, vol 4, chap 137, pp 3712, 3721-3722. Question #15 ANSWER=E ______________________________________________________________________________ Numerous organ systems may have late effects of treatment for childhood cancer. Stage III Wilms' tumor is defined as a residual nonhematogenous tumor confined to the abdomen. In this child, the tumor undoubtedly required nephrectomy, chemotherapy, and radiation therapy. Abdominal radiation confers a 12% incidence of ovarian failure that will increase up to 30% if combined with alkylating chemotherapeutic agents such as cyclophosphamide, isophosphamide, or cisplatin. Congestive heart failure occurs in 1.7% of doxorubicin patients. Pulmonary fibrosis would be rare, and is usually associated with either whole lung radiation or bleomycin administration. Second malignancies occur in

Page 8: Live Course Handout.cfm Second Part

about 1% of patients after ten years. The incidence of renal failure in unilateral Wilms' survivors is less than 0.2%. Stillman RJ, Schinfield JS, Schiff I, et al: Ovarian failure in long-term survivors of childhood malignancy. AM J OBSTET GYNECOL 1987;139:62.Ritchey ML, Shamberger RC: Pediatric urologic oncology, Wein, AJ, Kavoussi LR, Novick AC, Partin AW, Peters CA (eds): CAMPBELL-WALSH UROLOGY, ed 10. Philadelphia, Elsevier Saunders, 2012, vol 4, chap 137, pp 3722-3723. Question #16 ANSWER=C ______________________________________________________________________________ This tumor by ultrasound description is a testicular teratoma, which in a prepubertal patient is a universally benign tumor. They are encapsulated masses, thus making complete enucleation possible with salvage of the remaining testis tissue. In the referenced review, a total of 22 prepubertal boys presenting with teratomas had no evidence of tumor cells beyond the mass itself in any case, nor was there any evidence of multifocal tumors. No patient in long-term follow up demonstrated any evidence of tumor recurrence. An inguinal approach for a testis mass is warranted. Rushton HG, Bellman AB, Sesterhenn I, Patterson K, Mostofi FK: Testicular sparing surgery for prepubertal teratoma of the testis: A clinical and pathological study. J UROL 1990;144:726.Ritchey ML, Shamberger RC: Pediatric urologic oncology, Wein, AJ, Kavoussi LR, Novick AC, Partin AW, Peters CA (eds): CAMPBELL-WALSH UROLOGY, ed 10. Philadelphia, Elsevier Saunders, 2012, vol 4, chap 137, pp 3727-3728. Question #17 ANSWER=E ______________________________________________________________________________ The development of oligohydramnios in the third trimester combined with normal fetal urinary electrolytes and the absence of renal cysts and normal renal echogenicity suggest that Twin B's kidneys have a good prognosis. In this particular clinical situation, repeat aspiration of Twin B's bladder would put the normal twin A at risk for possible infection (chorioamnionitis) and sepsis. Fetal interventional surgery (vesicoamniotic shunt) is not indicated due to the third trimester presentation and risk of injury to the normal twin A. In view of serial fetal ultrasounds revealing worsening oligohydramnios and progressive bilateral hydroureteronephrosis of Twin B, it is most reasonable to proceed with measuring the lecithin-sphingomyelin ratio of Twin A and progress to early delivery if the ratio is normal. If the LS ratio reveals pulmonary immaturity, betamethasone (12 mg weekly) can be administered and the LS ratio reassessed. A strong reason for considering early delivery for this particular woman is to prevent progressive issues with Twin B during the final few weeks of pregnancy from affecting the normal outcome expected with Twin A. Coplen DE: Prenatal intervention for hydronephrosis. J UROL 1997;157:2270-2277. Dudley JA, Haworth JM, McGraw M, et al: Clinical relevance and implications of antenatal hydronephrosis. ARCH DIS CHILD 1997;76:31-34.Lee RS, Borer JG: Perinatal urology, Wein, AJ, Kavoussi LR, Novick AC,

Page 9: Live Course Handout.cfm Second Part

Partin AW, Peters CA (eds): CAMPBELL-WALSH UROLOGY, ed 10. Philadelphia, Elsevier Saunders, 2012, vol 4, pp 3062-3064. Question #18 ANSWER=C ______________________________________________________________________________ This child has sacral agenesis as a cause of his neurogenic bladder and bladder wall thickening, as seen on the KUB on the ultrasound. The VCUG reveals a severely trabeculated bladder with an open bladder neck and left VUR. The diagnosis of sacral agenesis can be confirmed by a lateral plain film. Due to the increased risk of tethered cord in patients with partial or complete sacral agenesis, an MRI scan of the spine is indicated. Many children with sacral agenesis will be picked up as infants due to the physical exam finding of flattened upper buttocks, leading to suspicion of the abnormality and subsequent evaluation. Management should focus on treatment of the concurrent neurogenic bowel and the results of urodynamic testing. MacLellan DL, Bauer SB: Neuropathic dysfunction of the lower urinary tract, Wein, AJ, Kavoussi LR, Novick AC, Partin AW, Peters CA (eds): CAMPBELL-WALSH UROLOGY, ed 10. Philadelphia, Elsevier Saunders, 2012, vol 4, chap 128, pp 3447-3448. Question #19 ANSWER=A ______________________________________________________________________________ This child has classic overactive bladder/dysfunctional elimination syndrome. Treatment should begin with antibiotic prophylaxis, bowel management, timed voiding, and maintenance of a bowel and voiding calendar. Antimuscarinics could worsen the constipation, and should only be added based upon response to the initial therapy. The presence of spina bifida occulta should not prompt a urodynamic evaluation nor neurosurgical consultation unless she does not respond to primary therapy. At this time, she does not need urodynamic investigation, MRI scans of the spine, or treatment of her VUR, as the symptoms and findings suggest simple overactive bladder with associated constipation. Yeung CK, Sihoe JDY: Non-neuropathic dysfunction of the lower urinary tract in children, Wein, AJ, Kavoussi LR, Novick AC, Partin AW, Peters CA (eds): CAMPBELL-WALSH UROLOGY, ed 10. Philadelphia, Elsevier Saunders, 2012, vol 4, chap 127, p 3411. Question #20 ANSWER=D ______________________________________________________________________________ Ureteroenteric strictures will occur at an incidence of approximately 0.5% per year if the anastomosis is freely refluxing and one percent per year if a tunneled ureteroenteric anastomosis was performed. This patient has developed a late occurring ureteroenteric stricture and should undergo excision of the anastomosis with a cuff of surrounding sigmoid colon. Patients who are converted from ureterosigmoidostomy to other forms of diversion or who are undiverted must have the region of ureterosigmoidostomies excised even if there is no present evidence of malignancy. Indeed, malignancy

Page 10: Live Course Handout.cfm Second Part

at the anastomotic site has been documented to occur decades after the anastomosis is defunctionalized. While all of the other options are definite treatment possibilities, the one component that is required is the excision of the anastomosis site with a cuff of sigmoid tissue. Husmann DA, Spence HM: Current status of tumor of the bowel following ureterosigmoidostomy: A review. J UROL 1990;144:607-610.Adams MC, Joseph DB: Urinary tract reconstruction in children, Wein, AJ, Kavoussi LR, Novick AC, Partin AW, Peters CA (eds): CAMPBELL-WALSH UROLOGY, ed 10. Philadelphia, Elsevier Saunders, 2012, vol 4, chap 129, pp 3462-3463, 3481-3482, 3492-3493. Question #21 ANSWER=E ______________________________________________________________________________ In premature infants, nephrocalcinosis (defined as calcium deposits within the medullary pyramids of the renal parenchyma) is predominantly seen in infants treated with furosemide for cardiopulmonary disorders. In premature and underweight infants, kidney function is significantly underdeveloped. Immature renal function is associated with low urinary citrate secretion, hypercalciuria, decreased bicarbonate resorptive activity, the latter resulting in alkaline urine pH. This chemical constituency of the urine and its alkalinity greatly predisposes the infant to calcium crystal precipitation. The use of loop diuretics to manage the fluid overload conditions associated with cardiopulmonary abnormalities will greatly aggravate the possibility of nephrocalcinosis by further increasing the urinary calcium excretion. Classically, premature neonates with nephrocalcinosis present with the finding of hematuria with a past medical history of prematurity or low birth weight associated with cardiopulmonary dysfunction, treated with furosemide therapy. For prognostic purposes, a urinary calcium creatinine ratio should be obtained at the time of diagnosis. If a ratio of <0.4 is found at diagnosis, the vast majority of patients will spontaneously resolve their stones upon immediate cessation of furosemide. The median time to resolution of nephrocalcinosis is approximately six months. Individuals with ratios >0.4 have been found to have increasing risks of developing symptomatic stones requiring surgical intervention, the risk directly related to higher ratios. The initial treatment is immediate cessation of furosemide treatment. Treatment may include changing to thiazide diuretic if needed for management of fluid overload and following up with serial ultrasounds every three to six months. Randomized studies have shown that switching to a thiazide diuretic does not actively cause the dissolution of calculi, and the initiation of a thiazide diuretic is predominantly for fluid management not for treatment of the nephrocalcinosis. Neither dietary calcium restriction nor acidification of the urine has been found to be beneficial, and alkalinization of the urine could actually be harmful aiding in the precipitation of further calcium crystals. Ferrandino MN, Pietrow PK, Preminger GM : Evaluation and medical management of urinary lithiasis, Wein, AJ, Kavoussi LR, Novick AC, Partin AW, Peters CA (eds): CAMPBELL-WALSH UROLOGY, ed 10. Philadelphia, Elsevier Saunders, 2012, vol 2, chap 46, p 1321.Pope JC, Trusler LA, Klein AM, et al: The natural history of nephrocalcinosis in premature infants treated with loop diuretics. J UROL 1996;156:709. Question #22 ANSWER=B ______________________________________________________________________________

Page 11: Live Course Handout.cfm Second Part

In patients with a renal stone within a calyceal diverticulum, PCNL is associated with minimal morbidity, and provides the patient with the best chance of becoming symptom and stone free, with the added benefit of possible ablation of the diverticular cavity. A direct percutaneous approach into the calyceal diverticulum is preferable because it allows use of a rigid nephroscope for stone extraction, dilation and incision of the diverticular neck (if desired), and fulguration of the diverticular epithelium. Retrograde ureteroscopic management is a reasonable option for select patients with diverticula in the upper and middle portions of the kidney and when the stone burden is less than 2 cm. Retrograde identification of the small communication between the collecting system and the diverticulum may be difficult or impossible. Ureteroscopy in prepubertal male children may also prove technically challenging and is associated with a higher morbidity with a one to three percent chance of developing a urethral or ureteral stricture on long-term follow-up. Laparoscopic caliceal diverticulectomy and stone extraction have been described, but a posterior calyx would likely require a retroperitoneal approach. This is technically more challenging, and the success and morbidity are comparable to the percutaneous approach. Open stone removal and caliceal diverticulectomy can be successfully performed with a stone free rate minimally higher than a percutaneous approach; however, it is associated with a higher morbidity and a longer hospital stay than a percutaneous approach. Matlaga BR, Lingeman JE: Surgical management of upper urinary tract calculi, Wein, AJ, Kavoussi LR, Novick AC, Partin AW, Peters CA (eds): CAMPBELL-WALSH UROLOGY, ed 10. Philadelphia, Elsevier Saunders, 2012, vol 2, chap 48, p 1357.Pearle MS, Lotan Y: Urinary lithiasis: Etiology, epidemiology, and pathogenesis, Wein, AJ, Kavoussi LR, Novick AC, Partin AW, Peters CA (eds): CAMPBELL-WALSH UROLOGY, ed 10. Philadelphia, Elsevier Saunders, 2012, vol 2, chap 45, p 1279. Question #23 ANSWER=D ______________________________________________________________________________ In patients with renal tubular acidosis, the urinary pH is inappropriately elevated in the presence of systemic acidosis. To buffer the metabolic acidosis, urinary citrate is reabsorbed, resulting in hypocitraturia. The elevated urinary pH and hypocitraturia will predispose the patient to the development of calcium phosphate stones. Documentation of normalization of urinary citrate levels while on potassium citrate supplementation is the most sensitive method used to document both the resolution of the metabolic acidosis and the therapeutic efficacy. Correction of the metabolic acidosis will result in a decrease in urinary calcium secretion and decreased citrate reabsorption. The combination of increased urinary citrate and reduced urinary calcium will result in a net reduction in the supersaturation of calcium phosphate and prevention of further stone formation. Ferrandino MN, Pietrow PK, Preminger GM : Evaluation and medical management of urinary lithiasis, Wein, AJ, Kavoussi LR, Novick AC, Partin AW, Peters CA (eds): CAMPBELL-WALSH UROLOGY, ed 10. Philadelphia, Elsevier Saunders, 2012, vol 2, chap 46, p 1315.Reddy PP and Minevich: Renal calculus disease, in Docimo): CLINICAL PEDIATRIC UROLOGY, ed 3, Philadelphia, WB Saunders Co, 1992, vol 2, chap 25, p 393.

Page 12: Live Course Handout.cfm Second Part

Question #24 ANSWER=C ______________________________________________________________________________ Paratesticular rhabdomyosarcoma arises in the distal portion of the spermatic cord and may invade the testes or surrounding tissue. Approximately 7% of all rhabdomyosarcomas are located in the paratesticular area. Presentation is bimodal with peak incidence between one and five years of age and a smaller peak around puberty. Radical inguinal orchiectomy is recommended for initial treatment. Paratesticular rhabdomyosarcomas have a high incidence of retroperitoneal lymphatic spread with 26% of patients in IRS-I and IRS-II found to have metastasis. It is recommended that all patients with paratesticular primary tumors have thin-cut, contrast enhanced abdominal and pelvic CT scans obtained to evaluate for nodal involvement. For patients who have Group I disease (i.e., tumor confined and completely resected), are younger than ten years, and in whom CT scans show no evidence of lymph node enlargement, retroperitoneal node biopsy/sampling is unnecessary. A repeat CT scan every three months for two years is recommended. For patients with suggestive or positive CT scans, retroperitoneal lymph node sampling (but not formal node dissection) is recommended, and treatment is based on the findings of this procedure. In children who are ten years of age or older, a staging ipsilateral retroperitoneal lymph node dissection is currently required to be placed on an IRS study protocol. Ipsilateral RPLND is preferred over a bilateral RPLND in an attempt to maintain ejaculation potential. The treatment recommended is based on the pathologic findings of the initial surgical pathology and retroperitoneal findings. Weiner ES, Lawrence W, Hays D, et al: Retroperitoneal node biopsy in childhood paratesticular rhabdomyosarcoma. J PED SUR 1994; 29:171-178.NCI, Childhood Rhabdomyosarcoma web site, updated Oct, 2014. www.cancer.gov/cancertopics/pdq/treatment/childrhabdomyosarcoma/patent. Ritchey ML, Shamberger RC: Pediatric urologic oncology, Wein, AJ, Kavoussi LR, Novick AC, Partin AW, Peters CA (eds): CAMPBELL-WALSH UROLOGY, ed 10. Philadelphia, Elsevier Saunders, 2012, vol 4, chap 137, pp 3704-3710. Question #25 ANSWER=D ______________________________________________________________________________ The grade of renal injury in this patient cannot yet be determined, and a repeat (delayed) scan ten to 20 minutes after contrast injection should be performed. The spiral CT scan is so rapid, that frequently the excreted contrast has not had time to make its way into the collecting system and through the entire upper tract. The ability to accurately grade a renal injury is especially hampered if there has been renal contusion or laceration. These latter injuries will almost invariably be associated with delayed excretion of contrast on the involved side, resulting in inaccurate staging of an injury unless a delayed film is ascertained. A retrograde ureteropyelogram would be diagnostic; however, it is invasive and requires anesthesia and its intended risks. Observation is inappropriate without having a diagnosis, and an additional MRI study would add significant cost. There are two absolute and one relative indication to obtain a CT cystogram to evaluate for a traumatic bladder injury. Absolute indications are gross hematuria in the presence of a pelvic fracture, and inability to void; it may be prudent to obtain a retrograde ureteropyelogram prior to a cystogram in the latter scenario. The relative indication for cystography is gross hematuria. Cystography should always be obtained by retrograde injection of contrast. This recommendation is due to the fact that omentum or small bowel may block a small

Page 13: Live Course Handout.cfm Second Part

intraperitoneal bladder rupture and seal the perforation, the leak only becoming apparent at the time of voiding, detrusor pressure pushing the obstructing visceral component out of the laceration site. To improve the diagnostic accuracy of cystography, retrograde flow of contrast is recommended over passive filling of the bladder by antegrade contrast. It is hypothesized that retrograde filling will push the visceral component out of the laceration site. The bladder should be filled to at least two-thirds of the estimated bladder capacity for age to assess for a bladder injury. Mulligan JM, Cagiannos I, Collins JP, et al: Ureteropelvic junction disruption secondary to blunt trauma: Excretory phase imaging (delayed films) should help prevent a missed diagnosis. J UROL 1998;159:67-70. Husmann DA: Pediatric genitourinary trauma, Wein, AJ, Kavoussi LR, Novick AC, Partin AW, Peters CA (eds): CAMPBELL-WALSH UROLOGY, ed 10. Philadelphia, Elsevier Saunders, 2012, vol 4, chap 138, pp 3731-3738. Question #26 ANSWER=A ______________________________________________________________________________ In neonates, approximately ten percent will void in the delivery room with 93% of infants voiding by 24 hours and 98% by 48 hours. Evaluation for fetal anuria or oligoria should not be instituted until after 48 hours have passed since birth. The most common reason for delayed voiding is unobserved void in the delivery room or poorly measured urine output at the time of meconium passage. The second most common etiology is hypovolemia, followed by posterior urethral valves, bilateral UPJ obstruction, neurogenic bladder, and renal dysplasia. The initial evaluation after 48 hours of neonatal anuria should be with a renal, bladder and spinal ultrasound. Serum creatinine and electrolytes can be obtained for a baseline but will usually represent maternal values. An ultrasound evaluation for a tethered cord is highly accurate in an infant until the third month of life, when ossification of the spinal vertebrae interfere with the technique and a spinal MRI scan becomes the test of preference. The conus medullaris of the spinal cord in an adolescent or adult should terminate between T12-L2, in neonates 98% will terminate above the L2-L3 interspace with < 2% extending to the upper level of L3. Findings of the termination of the conus at the mid L-3 or lower are consistent with concerns for a tethered cord. In infants with a history of anuria for 48 hours with normal renal, bladder and spinal ultrasounds, baseline electrolytes with creatinine should be obtained, and feeding encouraged. If the child remains with no urine output for an additional 24 hours, a repeat renal bladder ultrasound, electrolytes, and creatinine determination is recommended. MacLellan DL, Bauer SB: Neuropathic dysfunction of the lower urinary tract, Wein, AJ, Kavoussi LR, Novick AC, Partin AW, Peters CA (eds): CAMPBELL-WALSH UROLOGY, ed 10. Philadelphia, Elsevier Saunders, 2012, vol 4, chap 128, pp 3447-3448.Moore KL, Persaud TVN: The nervous system, in Moore KL, Persaud TVN (eds): THE DEVELOPING HUMAN: CLINICALLY ORIENTED EMBRYOLOGY, ed 5. Philadelphia, WB Saunders, 1993, chap 18, pp 385-422. Question #27 ANSWER=C ______________________________________________________________________________

Page 14: Live Course Handout.cfm Second Part

Persistent Mullerian duct syndrome is one of the 46 XY disorders of sexual differentiation. Classically, the patient will have a 46 XY genotype, with bilateral undescended testicles with either Fallopian tubes, or on occasion the uterus being found within the hernia sac. This finding gives this syndrome its pseudonym of Hernia Uteri Inguinalis. Persistent Mullerian duct syndrome is inherited in an autosomal recessive pattern and is caused by the deficient action of fetal anti-Müllerian hormone (AMH). This deficient action may be due to mutations of the AMH gene, the anti-Müllerian hormone receptor, or insensitivity of the target organs to AMH. AMH is produced by the Sertoli cells during the seventh week of gestation with regression of the Mullerian ducts occurring during the eighth to ninth week. Orchidopexy may be problematic in these patients with the testicular blood supply and the vas deferens intrinsically associated with the Mullerian remnants. Attempts to excise the Mullerian structures will frequently lead to damage to the vas deferens and gonadal blood supply. Orchidopexy is greatly facilitated by detachment of the salpinx from the uterus and mobilization of the cord and salpinx into the inguinal canal. Although there is hypothetical concern that retention of the Mullerian structures could lead to malignant transformation, this hypothesis has little to no published support. Vigorous attempts to remove Mullerian structures should be pursued only if clinical symptoms warrant their excision. Infertility with azoospermia is found in the vast majority of these patients. Almost all individuals with this disorder develop hypotestosteronemia in the third or fourth decade of life, requiring hormonal supplementation. Testicular feminization is the extreme form of androgen insensitivity (a type of 46 XY, DSD). Approximately 90-95% of individuals with this disorder present with complaints of primary amenorrhea in a phenotypic appearing female, and approximately five percent present with testes located in a hernia sac at the time of an inguinal hernia repair. In androgen insensitivity, there are no Mullerian structures, as MIF production and function is normal. Neither mixed nor pure gonadal dysgenesis would have normal appearing gonads bilaterally. Ovotesticular disorder requires the presence of both ovarian and testicular tissue, and, therefore, would not have normal appearing testes. Belville C, Josso N, Picard JY: Persistence of Mullerian derivatives in males. AM J MED GENET 1999;89:218-223.Souto CA, Oliveira MD, Teloken C, et al: Persistence of Mullerian duct derivative syndrome in two male patients with bilateral cryptorchidism. J UROL 1995;153:1637-1638.Diamond DA, Yu RN: Sexual differentiation: Normal and abnormal, Wein, AJ, Kavoussi LR, Novick AC, Partin AW, Peters CA (eds): CAMPBELL-WALSH UROLOGY, ed 10. Philadelphia, Elsevier Saunders, 2012, vol 4, chap 133, pp 3624-3625. Question #28 ANSWER=D ______________________________________________________________________________ In general, nuclear renography in patients with normal renal function and a duplication anomaly have found that the lower pole of a kidney will supply two-thirds of the ipsilateral renal function. For example, if normal ipsilateral renal function is 50%, two-thirds or 33% of the total renal function should come from the lower pole. Given this patient's diminished function, prolonged washout times, and history of UTI, intervention is indicated. Since the lower pole segment has >10% function, the best surgical alternative is a lower to upper pole pyeloureterostomy to relieve the obstruction and provide an opportunity for renal recoverability. If function of <10% is seen, options are for performing either a lower pole pyelostomy and evaluate for recoverability during the important first year of nephrogenesis or alternatively, if significant areas of renal dysplasia are noted at time of surgery, a lower pole partial nephrectomy could be performed.

Page 15: Live Course Handout.cfm Second Part

Peters CA, Schlussel RN, Mendelsohn C: Ectopic ureter, ureterocele, and ureteral anomalies, Wein, AJ, Kavoussi LR, Novick AC, Partin AW, Peters CA (eds): CAMPBELL-WALSH UROLOGY, ed 10. Philadelphia, Elsevier Saunders, 2012, vol 4, chap 121, p 3261. Question #29 ANSWER=C ______________________________________________________________________________ Prolonged follow-up in children with posterior urethral valves reveals that there is a natural progression in bladder dynamics with age. Specifically, improvement in both detrusor compliance and a decrease in overactivity occur with maturity. Indeed, a number of patients with previously poorly compliant or overactive bladders in infancy and childhood will develop decreased bladder sensation and myogenic bladder failure in early adulthood. Knowledge of the alterations that occur in the bladder dysfunction has resulted in the current rationale for treating urinary incontinence, high bladder pressures, and worsening hydronephrosis in patients with PUV in a conservative fashion, e.g., antimuscarinics, timed voiding, overnight catheter drainage. It is currently considered prudent to wait until the postpubertal time period to consider the patient for bladder augmentation in this patient population. Casale AJ: Posterior urethral valves, Wein, AJ, Kavoussi LR, Novick AC, Partin AW, Peters CA (eds): CAMPBELL-WALSH UROLOGY, ed 10. Philadelphia, Elsevier Saunders, 2012, vol 4, chap 126, pp 3402-3403. Question #30 ANSWER=C ______________________________________________________________________________ Bladder exstrophy, cloacal exstrophy, and epispadias are all variants of the exstrophy-epispadias complex. The cause of this complex is thought to be the failure of the lateral mesodermal folds to grow into the cloacal membrane. Specifically, at four weeks of gestation, the cloacal membrane is located inferior to the umbilicus. It is the anterior covering for the common cloaca, and is composed of only two components, the endoderm and ectoderm. During the fourth to eighth weeks of gestation, the cloacal membrane is altered by the ingrowth of mesoderm between its two components. The mesoderm arises from the lateral mesodermal folds that surround the cloacal membrane. The mesoderm will migrate medially into the cloacal membrane in a cephalad to caudal direction. The ingrowth of mesoderm will result in the progressive displacement of the cloacal membrane to the caudal region. As the lateral mesodermal folds begin to join in the midline, the urorectal septum is simultaneously growing from a cephalad to caudal direction. The urorectal septum will be dividing the common cloaca into the anterior urogenital sinus and the posterior anorectal canal. The urorectal septum will eventually reach the caudally displaced cloacal membrane. When the urorectal septum meets the caudally displaced cloacal membrane, the perineal tissues are formed. Development of the perineum will result in the separation of the cloacal membrane into two separate areas; the anterior urogenital sinus and posterior anal canals by the eighth week of gestation. Exstrophy epispadias complex hypothetically occurs due to either the abnormal migration or midline fusion failure of the mesodermal folds or, alternatively, a loss of the cloacal membrane due to a fetal vascular accident. The timing of when the fusion failure or vascular accident occurs results in the different spectrums of this disorder. Cloacal exstrophy occurs when there

Page 16: Live Course Handout.cfm Second Part

is incomplete advancement of the urorectal septum to meet the cloacal membrane. This will result in a failure to separate the hind gut from the urogenital sinus, leading to persistence of a cloaca. Classical exstrophy occurs when there is failure of the anterior mesodermal ingrowth; however, the urorectal septum still meets the caudal cloacal membrane resulting in formation of the anorectal tube and the perineal body formation. Epispadias occurs when the most distal part of the anterior portion of the cloaca membrane fails to have midline mesodermal fusion. A common cloaca occurs when the urorectal septum fails to meet the cloacal membrane to form the perineal body. A common urogenital sinus occurs when the anterior urogenital sinus fails to divide into its two components. Purves JT, Gearhart JP: The bladder exstrophy - epispadias - cloacal exstrophy complex, in Gearhart JP, Rink RC, Mouriquand PDE (eds): PEDIATRIC UROLOGY. Philadelphia, Saunders Elsevier, 2010, chap 30, p 411. Question #31 ANSWER=C ______________________________________________________________________________ With normal development, the ureteral bud should separate from the common excretory duct and become incorporated into the trigone of the bladder. When there is failure of separation between the ureteral bud and common excretory duct, the trigone and bladder neck fail to develop, resulting in a small bladder capacity, incompetent bladder neck, and bilateral ureteral ectopy. This occurs when the ureteral buds originate more cranially than normal on the mesonephric duct. Laterally-positioned refluxing ureters are due to ureteral buds which originate too caudally from the mesonephric duct. Ureteral duplication is due to two separate ureteral buds originating from the same mesonephric duct. Ureteroceles are hypothesized to be due to obstruction of the distal ureter due to failure of the distal ureteral orifice to recanalize during ureteral development. Renal ectopia is thought to be due to aberrant blood supply to the developing renal unit. Cuckow PM, Nyirady P: Embryology and pathophysiology of the kidneys and urinary tracts, in Gearhart JP, Rink RC, Mouriquand PDE (eds):Pediatric Urology. Philadelphia, W.B. Saunders, 2001, chapter 1, p 8.Peters CA, Schlussel RN, Mendelsohn C: Ectopic ureter, ureterocele, and ureteral anomalies, Wein, AJ, Kavoussi LR, Novick AC, Partin AW, Peters CA (eds): CAMPBELL-WALSH UROLOGY, ed 10. Philadelphia, Elsevier Saunders, 2012, vol 4, chap 121, p 3236. Question #32 ANSWER=C ______________________________________________________________________________ Classically, patients with Tuberous Sclerosis (TS) will be said to have facial dermatologic lesions (angiofibromas, hypomelanotic patches) developmental delay, and seizures. Not all patients with Tuberous Sclerosis (TS) will manifest this classic triad, and the diagnosis may be made when two of the following findings are noted and then confirmation of the diagnosis by genetic screening. The findings associated with TS are: facial dermatologic lesions (angiofibromas, hypomelanotic patches), retinal hamartomas, cortical tubers of the brain, subependymal nodules, subependymal astrocytoma, cardiac rhabdomyosarcoma, lymphangiomyomatosis, and renal angiomyolipomas. Renal angiomyolipomas are estrogen sensitive, and may rapidly grow at the time of puberty or involute with onset of menopause.

Page 17: Live Course Handout.cfm Second Part

They will classically present with spontaneous retroperitoneal bleeding during these time periods. Both renal cell carcinoma and malignant angiomyolipoma are concerns in patients with TS and may present in childhood. TS is not associated with hepatic fibrosis, thyroid cancer, or cerebellar hemangiomas. Chronic renal failure generally does not occur in TS. There are patients, however, with disruption of both autosomal polycystic kidney gene (PKD1) and TS genes. These patients will have more severe cystic renal disease, and are at higher risk for hypertension and renal failure. Greenbaum L: Cystic kidney disease, in Docimo SG, Canning DA, Khoury AE (eds): CLINICAL PEDIATRIC UROLOGY, ed 5. London, Informa Healthcare, 2007, chap 22, pp 344-346.Pope JC IV: Renal dysgenesis and cystic disease of the kidney, Wein, AJ, Kavoussi LR, Novick AC, Partin AW, Peters CA (eds): CAMPBELL-WALSH UROLOGY, ed 10. Philadelphia, Elsevier Saunders, 2012, vol 4, chap 118, p 3179. Question #33 ANSWER=C ______________________________________________________________________________ This child has Mayer-Rokitansky syndrome, which is the partial or complete absence of the vagina, uterus, and fallopian tubes. Her ovaries are normal. Classically, these patients present for evaluation with primary amenorrhea; however, up to 30% of patients will have unilateral renal agenesis or developmental renal anomalies. Twenty percent of the patients will have spinal anomalies (thoracic or lumbar scoliosis), with up to two-thirds of these individuals having manifestations of a neurogenic bladder. In patients with Mayer-Rokitansky syndrome and concurrent spinal anomalies, the presence of a tethered cord and assessment for the presence of a neurogenic bladder by MRI and urodynamic studies should be performed. In addition to the renal and spinal abnormalities, an additional 15% of patients will be found to have either congenital deafness or cardiac anomalies. Aniridia, coarctation of the aorta, inguinal hernia, and malrotation of the bowel are not associated with this disorder. Rink RC, Kaefer M: Surgical management of disorders of sexual differentiation, cloacal malformation, and other abnormalities of the genitalia in girls, Wein, AJ, Kavoussi LR, Novick AC, Partin AW, Peters CA (eds): CAMPBELL-WALSH UROLOGY, ed 10. Philadelphia, Elsevier Saunders, 2012, vol 4, chap 134, pp 3632-3633. Question #34 ANSWER=E ______________________________________________________________________________ In a patient with a complete C-4 injury, CIC is impossible due to limited hand function and no ability for self-transfer. Augmentation with an ileocystoplasty or an appendicovesicostomy alone would face the same CIC problem. Due to the limited bladder capacity, catheterization by a surrogate would not correct the issues of incontinence and a decubitus ulcer. Ileal conduit has a worse long-term prognosis than an indwelling suprapubic tube due to the increased risk of stomal or ureteroenteric obstruction. In this scenario, suprapubic tube management has the best prognosis. Atan A, Konety BR, Nangia A, Chancellor MB: Advantages and risks of ileovesicostomy for the management of neuropathic bladder. Urology 1999;54:636?640. Adams MC, Joseph DB: Urinary tract

Page 18: Live Course Handout.cfm Second Part

reconstruction in children, Wein, AJ, Kavoussi LR, Novick AC, Partin AW, Peters CA (eds): CAMPBELL-WALSH UROLOGY, ed 10. Philadelphia, Elsevier Saunders, 2012, vol 4, chap 129, pp 3457-3497.MacLellan DL, Bauer SB: Neuropathic dysfunction of the lower urinary tract, Wein, AJ, Kavoussi LR, Novick AC, Partin AW, Peters CA (eds): CAMPBELL-WALSH UROLOGY, ed 10. Philadelphia, Elsevier Saunders, 2012, vol 4, chap 128, pp 3454-3455. Question #35 ANSWER=D ______________________________________________________________________________ The stages of renal failure, stage 1, normal renal function, GFR >90 ml/min/BSA, stage 2, 89-60 ml/min/BSA, stage 3, 59-30 ml/min/BSA, stage 4, 29-15 ml/min/BSA, stage 5, <15 ml/min/BSA. Titration of renal secreted agents should be based on renal function and reaches a critical threshold for most renal secreted agents in stage 3 renal failure. Caution should therefore be given in prescribing antibiotics in any patient with grade 3 renal failure or higher. Ceftazidime, ciprofloxacin, sulfamethoxazole-trimethoprim, and nitrofurantoin are all secreted in part by the kidney. These require adjustment of either dose or interval in the face of chronic renal insufficiency. Drugs that are excreted primarily by the liver such as ceftriaxone do not require adjustment unless there is coexisting liver failure. Sulfamethoxazole and nitrofurantoin are examples of drugs with low and likely inadequate therapeutic urine concentrations in patients with creatinine clearances of <50 ml/min, stage 3 renal failure. Urine concentrations of ciprofloxacin, levofloxacin, ceftazidime remain therapeutic as renal function fails; however, systemic toxicity of these agents reach critical threshold with clearances of <20 ml/min/BSA, stage 4 and 5 renal failure. Therapeutic dosages of all quinolones and ceftazidime are recommended to be cut by 50% in stage 4 and 5 renal failure. Schaeffer AJ, Schaeffer EM: Infections of the urinary tract, Wein, AJ, Kavoussi LR, Novick AC, Partin AW, Peters CA (eds): CAMPBELL-WALSH UROLOGY, ed 10. Philadelphia, Elsevier Saunders, 2012, vol 1, chap 10, p 257.www.uphs.upenn.edu/bugdrug/antibiotic_manual/renal.htmwww.globalrph.com/renaldosing2.htm Question #36 ANSWER=E ______________________________________________________________________________ Approximately 15% of patients with a common urogenital sinus or cloacal anomaly are referred to the urologist with the misdiagnosis of an intersex anomaly. The ambiguous appearance of the genitalia, combined with an imperforate anus, and the ultrasound appearance of three fluid-filled structures converging in the pelvis are all consistent with a persistent cloaca. There is often obstructive uropathy related to either a concurrent neurogenic bladder (tethered cord), or more commonly due to pressure on the bladder outlet by the fluid-filled vagina (hydrocolpos). The hydrocolpos is secondary to poor egress of fluid from the narrowed luminal opening of the common urogenital sinus. Mayer-Rokitansky syndrome is associated with partial or complete agenesis of the salpinx, uterus, or vagina, and rarely presents in infancy with the most common presentation in late adolescence or early adulthood with primary amenorrhea. Variants of the adrenogenital syndrome, or congenital adrenal hyperplasia, leads to genital ambiguity with a normal rectal opening.

Page 19: Live Course Handout.cfm Second Part

Rink RC, Kaefer M: Surgical management of disorders of sexual differentiation, cloacal malformation, and other abnormalities of the genitalia in girls, Wein, AJ, Kavoussi LR, Novick AC, Partin AW, Peters CA (eds): CAMPBELL-WALSH UROLOGY, ed 10. Philadelphia, Elsevier Saunders, 2012, vol 4, chap 134, p 3649. Question #37 ANSWER=A ______________________________________________________________________________ The only tumor that fits the profile of this patient is a neuroblastoma stage IV-S. Stage IV-S is a metastatic disease state that has a favorable survival in infants with the majority of these tumors undergoing spontaneous regression. Prognostic factors which may adversely affect spontaneous regression include N-Myc amplification and DNA diploid tumors. Interestingly, DNA-aneuploidy has been shown to paradoxically predict a benign biologic behavior in stage IV -S patients, with diploid tumors associated with disease progression. Sites of metastatic involvement (liver, skin, or bone marrow) and surgical resection of the primary tumor have not been shown to adversely affect survival. Ritchey ML, Shamberger RC: Pediatric urologic oncology, Wein, AJ, Kavoussi LR, Novick AC, Partin AW, Peters CA (eds): CAMPBELL-WALSH UROLOGY, ed 10. Philadelphia, Elsevier Saunders, 2012, vol 4, chap 137, pp 3699-3701. Question #38 ANSWER=A ______________________________________________________________________________ Persistent dilation after a successful pyeloplasty is commonly seen. Indeed, the rate of demonstrable improvement in hydronephrosis may be extremely slow to nonexistent. Similarly, the extent of differential function expressed on nuclear renography most often remains essentially unchanged despite successful pyeloplasty. Minor variations in renal function can be seen in serial nuclear scans, and may reflect slight differences in either the technique or the designated regions of interest. In this patient with essentially unchanged hydronephrosis and stable renal function, it is most likely the patient has a satisfactory outcome from the pyeloplasty, and a repeat renal ultrasound at three months should be recommended. A Whitaker test, stent placement, endopyelotomy, and re-operative pyeloplasty all require anesthesia, are invasive and are only indicated for development of symptoms, deterioration of renal function, or for asymptomatic severe hydronephrosis that fails to show any improvement after more than six to 12 months of observation. Fung LCT, Lakshmanan Y: Anomalies of the renal collecting system: Ureteropelvic junction obstruction (pyelocaliectasis) and infundibular stenosis, in Belman AB, King LR, Kramer SA (eds): CLINICAL PEDIATRIC UROLOGY, ed 4. London, Marin Dunitz, 2002, chap 20, pp 559?631. Carr MC, Casale P: Anomalies and surgery of the ureter in children, Wein, AJ, Kavoussi LR, Novick AC, Partin AW, Peters CA (eds): CAMPBELL-WALSH UROLOGY, ed 10. Philadelphia, Elsevier Saunders, 2012, vol 4, chap 120, pp 3226-3228.

Page 20: Live Course Handout.cfm Second Part

Question #39 ANSWER=E ______________________________________________________________________________ Henoch-Schönlein is an idiopathic vasculitic syndrome observed primarily in boys between the ages of two and 11 years of age. It is manifested by nonthrombocytopenic purpura, colicky abdominal pain, arthralgia, acute scrotal pain with swelling (seen in 35% of patients), and glomerulonephritis. Renal involvement (hematuria) occurs in up to 50% of patients. Testicular ultrasound will reveal areas of vasculitis with both increased flow and decreased flow within the same testicle. Rarely will long-term testicular atrophy develop. No specific treatment is available for Henoch-Schönlein purpura and non-steroidal anti-inflammatories are recommended to treat the patient's symptoms. Although steroids and immunosuppressants have been used in patients with severe arthralgia, abdominal colic associated with gastrointestinal hemorrhage or glomerulonephritis, case-controlled studies have not shown them to be of any a proven benefit over non-steroidal anti-inflammatories. In essence, Henoch-Schonlein is usually a self-limited disorder, treated by serial observation and NSAIDS. Palmer LS, Trachtman H: Renal functional development and diseases in children, Wein, AJ, Kavoussi LR, Novick AC, Partin AW, Peters CA (eds): CAMPBELL-WALSH UROLOGY, ed 10. Philadelphia, Elsevier Saunders, 2012, vol 4, chap 112, p 3013.Barthold JS: Abnormalities of the testes and scrotum and their surgical management, Wein, AJ, Kavoussi LR, Novick AC, Partin AW, Peters CA (eds): CAMPBELL-WALSH UROLOGY, ed 10. Philadelphia, Elsevier Saunders, 2012, vol 4, chap 132, p 3594. Question #40 ANSWER=B ______________________________________________________________________________ Adrenal rest tissue is defined as ectopic adrenal cortical tissue classically found along the path of gonadal descent. Adrenal rest tissue is routinely found along the coeliac axis, within the broad ligament, spermatic cord, ovary or testis. It is present in up to 50% of normal newborns. The rest tissue is rarely >5 mm, and is usually noted as an incidental finding at surgery or autopsy. Hyperplasia of testicular, ovarian and paragonadal adrenal rest tissue in children with CAH is due to excessive ACTH stimulation and is most commonly found in patients with the salt-losing variant of 21-OHD with poor therapeutic control. The incidence of hyperplasic adrenal rest tissue detected by pelvic or testicular ultrasound within the gonads of patients with CAH is approximately 25%. On ultrasound the gonadal tumors tend to be bilateral with multiple foci within the gonad, they may, however, as in this case appear as a single gonadal mass. The gonadal adrenal rest tissue will substantially regress in up to 75% of patients after the individual reaches therapeutic levels of hormonal replacement. Initial treatment of a gonadal mass in a patient with congenital adrenal hyperplasia is to increase glucocorticoid therapy. MRI scan of the abdomen and pelvis should be considered if the mass fails to resolve following increased glucocorticoid therapy, with subsequent percutaneous biopsy, excisional resection, or oophorectomy performed as indicated. Vaughan ED Jr, Blumenfeld JD, Del Pizzo J, Schichman SJ, Sosa RE: The adrenals, in Walsh PC, Retik AB, Vaughan ED Jr, Wein AJ (eds): CAMPBELL'S UROLOGY, ed 8. Philadelphia, WB Saunders Co, 2002, vol 4, chap 101, p 3507.Tiosano D, Vlodavsky E, Filmar S, et al: Ovarian adrenal rest tumor in a

Page 21: Live Course Handout.cfm Second Part

congenital adrenal hyperplasia patient with adrenocorticotropin hypersecretion following adrenalectomy. HORM RES PAEDIATR 2010;74:223-228. Question #41 ANSWER=A ______________________________________________________________________________ Congenital adrenal hyperplasia is the most common cause of ambiguous genitalia at birth followed by mixed gonadal dysgenesis (45X/46XY). Interestingly, when the 45X/46XY karyotype is found on a screening amniocentesis, 90% to 95% of affected infants will have normal appearing male genitalia with descended testes at birth. Indeed, the presence of ambiguous genitalia or an undescended testicle within the 45x/46XY individuals being the rarity, not the norm. In patients with a 45X/46XY karyotype presenting with normal genitalia, approximately 25% of these patients will be found to have abnormal gonadal histology and/or function. Classically, these later patients present either at puberty or during the post-pubertal time period with either primary gonadal dysfunction with delayed sexual maturation or male infertility. Diamond DA, Yu RN: Sexual differentiation: Normal and abnormal, Wein, AJ, Kavoussi LR, Novick AC, Partin AW, Peters CA (eds): CAMPBELL-WALSH UROLOGY, ed 10. Philadelphia, Elsevier Saunders, 2012, vol 4, chap 133, p 3611. Question #42 ANSWER=E ______________________________________________________________________________ In this asymptomatic patient, the most likely diagnosis is an abdominal pseudocyst, as the ventriculoperitoneal shunt tubing appears to be within the cystic cavity. Peritoneal adhesions or loculation can lead to a CSF-filled pseudocyst around the abdominal portion of any ventriculoperitoneal shunt. If this patient had symptoms of abdominal pain or findings of abdominal tenderness or decreased CIC volumes, then CT cystogram would be indicated to look for possible bladder rupture/leak with urine secondarily collecting in the area of the ventriculoperitoneal shunt (by coincidence). As this patient is asymptomatic, however, there is very low suspicion for bladder leak. The most appropriate initial approach is neurosurgical consultation for consideration of revision of the shunt. MacLellan DL, Bauer SB: Neuropathic dysfunction of the lower urinary tract, Wein, AJ, Kavoussi LR, Novick AC, Partin AW, Peters CA (eds): CAMPBELL-WALSH UROLOGY, ed 10. Philadelphia, Elsevier Saunders, 2012, vol 4, chap 128, pp 3484-3485. Question #43 ANSWER=A ______________________________________________________________________________ The tracing shows an appropriate safe urinary storage zone; that is, detrusor pressures of 25 cm at the average catheterized bladder capacity and/or maximum estimated bladder capacity for age. Estimated capacity at one year of age is 90 ml (per Koff’s formula: age in years + 2 x 30 ml = estimated bladder capacity in ml). Although the patient does have a non-compliant bladder, detrusor storage pressures do

Page 22: Live Course Handout.cfm Second Part

not reach 40 cm H2O until 180 ml, well after her estimated bladder capacity and average catheterized bladder volumes are surpassed. Prior studies have shown that patients who keep catheterized storage bladder pressures consistently <35-40 cm H2O will have a low probability for upper tract deterioration over time. Thus, none of the interventions mentioned are to be recommended at this time. MacLellan DL, Bauer SB: Neuropathic dysfunction of the lower urinary tract, Wein, AJ, Kavoussi LR, Novick AC, Partin AW, Peters CA (eds): CAMPBELL-WALSH UROLOGY, ed 10. Philadelphia, Elsevier Saunders, 2012, vol 4, chap 128, pp 3437-3440. Question #44 ANSWER=A ______________________________________________________________________________ This neonate has a megalourethra. This congenital abnormality is most commonly seen in conjunction with Prune Belly Syndrome, VACTERL syndrome, and megacystis microcolon disorders. The etiology of this birth defect is currently unknown. Classically megalourethra exists in two types; however, it should be noted that this is a spectrum disorder that may present anywhere between the two classic forms. The fusiform subtype is defined as a megalourethra associated with absence of both the corpus cavernosum and spongiosum. The scaphoid subtype is associated with absence of the corpus spongiosum only. The fusiform subtype is more severe in nature and is associated with a 70% incidence of fetal or neonatal death. Death within these patients is almost invariably due to the megalourethra coexisting with severe urethral hypoplasia or atresia. The underdevelopment of the urethra will result in urinary tract obstruction and is associated with oligohydramnios, renal dysplasia, and poor pulmonary development. Death is usually directly related to pulmonary hypoplasia. The scaphoid subtype is less severe and is associated with a ten percent incidence of fetal death. Death in this subtype is usually due to concurrent renal dysplasia and pulmonary hypoplasia. In patients successfully delivered, up to 85% will have concurrent congenital malformations, necessitating an appropriate multisystem evaluation. From a genitourinary perspective, two-thirds of live born infants will be found to have a concurrent bladder abnormality (patent urachus, vesicoureteral reflux, bladder diverticulum, underactive bladder), and one-third will have co-existing renal anomalies (dysplasia, UPJ obstruction). All surviving fusiform subtypes have erectile dysfunction and most will require a penile prosthesis at adulthood. The treatment of megalourethra may be either a one or multi-staged urethroplasty depending on the co-existence of urethral hypoplasia. Biliary atresia, single ventricle, posterior urethral valves, and autosomal recessive polycystic kidney disease have not been associated with this disorder. It is noteworthy, however, that partial agenesis of the corpus spongiosum may give the radiographic picture similar to anterior urethral valves, the latter of which have, on rare occasions, been found in conjunction with this birth defect. Caldamone AA, Woodard JR: Prune-belly syndrome, Wein, AJ, Kavoussi LR, Novick AC, Partin AW, Peters CA (eds): CAMPBELL-WALSH UROLOGY, ed 10. Philadelphia, Elsevier Saunders, 2012, vol 4, chap 123, p 3314.Amsalem H, Fitzgerald B, Keating S: Congenital megalourethra: Prenatal diagnosis and postnatal/autopsy findings. ULTRASOUND OBSTET GYNECOL 2011;31(6):678-683. Question #45 ANSWER=D ______________________________________________________________________________

Page 23: Live Course Handout.cfm Second Part

Insufficiently concentrated urine in the presence of hypernatremia is consistent with diabetes insipidus. This child has nephrogenic diabetes insipidus with an obligatory polyuria secondary to tubular injury (congenital obstructive uropathy) with a urine volume of over two liters per day. He is volume contracted. The best way to normalize the sodium and treat the volume contraction is to give additional free water. D5 1/4 NS at 100 ml/hr will give free water and keep up with his voluminous urine output. Furosemide may cause further volume contraction without significant natriuresis. Nephrogenic diabetes insipidus is a tubular defect which does not allow ADH to bind to the vasopressin receptor in the collecting tubules. Therefore, DDAVP will not correct the situation. Palmer LS, Trachtman H: Renal functional development and diseases in children, Wein, AJ, Kavoussi LR, Novick AC, Partin AW, Peters CA (eds): CAMPBELL-WALSH UROLOGY, ed 10. Philadelphia, Elsevier Saunders, 2012, vol 4, chap 112, pp 3015-3016. Question #46 ANSWER=B ______________________________________________________________________________ A perivesical fluid collection that occurs this long after transplantation will usually be due to an obstructing lymphocele; however, a urinary leak from a delayed ureteral necrosis is a possibility. The perivesical fluid collection should be aspirated and creatinine values evaluated. Treatment is based on the analysis of the fluid. If no elevation in creatinine is noted, the diagnosis of a lymphocele is confirmed. Laboratory and ultrasound follow-up is recommended in three to five days to verify whether or not the lymphatic fluid continues to accumulate. Laparoscopic marsupialization of the lymphocele is recommended if the fluid recurs. If the fluid reveals an elevation in creatinine, the most likely diagnosis is delayed ureteral necrosis; ureteral stent placement is usually attempted. Success with this approach is limited, and most patients will require placement of a percutaneous nephrostomy tube. A MAG-3 renal scan can reveal an obstruction or a urinary leak, but does not offer the benefit of possibly treating the underlying pathology by aspiration of the obstructing lymphocele. Barry JM, Conlin MJ: Renal transplantation, Wein, AJ, Kavoussi LR, Novick AC, Partin AW, Peters CA (eds): CAMPBELL-WALSH UROLOGY, ed 10. Philadelphia, Elsevier Saunders, 2012, vol 2, chap 44, pp 1249-1250. Question #47 ANSWER=D ______________________________________________________________________________ Hemorrhagic cystitis will occur in 20-25% of bone marrow transplantation recipients, and in two to three percent of solid organ transplantation recipients. Hemorrhagic cystitis in the post-bone marrow transplant was once considered a complication of the myeloablative chemotherapy. Specifically, acrolein, one of the metabolic breakdown products of cyclophosphamide, was frequently listed as the stimulating etiology. With the routine use of hydration, Mesna, and improved viral detection methods, it is currently relevant to note that 90-95% of all cases of hemorrhagic cystitis within an immunocompromised patient are due to viral infections. Polyoma BKA virus is responsible for 80% of the infections with adenovirus responsible for the remaining 10-15%. Adenoviral cystitis usually has a benign self-limiting course that will rarely need treatment. In contrast, polyoma BKA viremia (in

Page 24: Live Course Handout.cfm Second Part

addition to the hemorrhagic cystitis) can cause interstitial nephritis, progressive nephropathy, and renal failure, and has been directly related to the development of ureteral strictures and detrusor noncompliance. Current treatment of hemorrhagic cystitis in the immunocompromised host is evaluation for viral cystitis (by evaluating the urine for viral titers by PCR analysis or viral cultures), I.V. hydration, correction of coagulopathy, and bladder irrigation to maintain the patient in a clot free status. Cidofovir intravesical instillation, 5 mg/kg in 60 ml of saline instillation can be used for one hour once a week. Cidofovir can also be given I.V. at a similar dose; however, bladder instillation may avoid nephrotoxic complications of the drug. Of note, cidofovir is active against both BKV and adenovirus, the two most likely viral agents to cause hemorrhagic cystitis. Paduch DA; Viral lower urinary tract infections. CURR UROL REPORTS 2007;8:324-335.Barry JM, Conlin MJ: Renal transplantation, Wein, AJ, Kavoussi LR, Novick AC, Partin AW, Peters CA (eds): CAMPBELL-WALSH UROLOGY, ed 10. Philadelphia, Elsevier Saunders, 2012, vol 2, chap 44, p 1226. Question #48 ANSWER=A ______________________________________________________________________________ In children with mixed gonadal dysgenesis, the risk of a gonadoblastoma developing in a streak gonad is approximately 25%. It is therefore recommended that all patients with mixed gonadal dysgenesis should have streak gonads removed. Gonadoblastoma is a tumor of low malignant potential, and a gonadectomy is adequate for treatment. Children raised as males are at a higher than normal risk for the development of a testicular seminoma in adulthood. At one time, testicular biopsy of the descended testicle at puberty was recommended to rule-out CIS and assess the risk for malignant transformation. If CIS was present, the testicle was removed. If the biopsy was normal, annual testicular ultrasounds were recommended for follow-up. This practice has not been proven to be of benefit and should be abandoned. It is currently recommended that the patient perform monthly self-testicular examination. Ritchey ML, Shamberger RC: Pediatric urologic oncology, Wein, AJ, Kavoussi LR, Novick AC, Partin AW, Peters CA (eds): CAMPBELL-WALSH UROLOGY, ed 10. Philadelphia, Elsevier Saunders, 2012, vol 4, chap 137, p 3729. Question #49 ANSWER=C ______________________________________________________________________________ The transverse colon conduit is the bowel segment of choice in patients who have received extensive pelvic radiation. Use of irradiated bowel is not recommended due to the radiation induced increased bowel wall fragility, vasculitis, and slower wound healing; all are factors known to be associated with radiation enteritis. Use of the ileum for urinary diversion in patients with a history of extensive pelvic radiation is associated with the risk of anastomotic leak (ureteroenteric or entero-entero) and vascular complications of the loop. Radiation-induced vasculitis and vascular fragility are manifest in either the acute post-operative time period one to five days post-surgery or in a delayed fashion with increased incidence of stomal or mid-loop stenosis or ureteroenteric strictures developing in the first five years of follow-up. Acute problems occurring during the first five days are usually due to either enteric vascular

Page 25: Live Course Handout.cfm Second Part

compromise or failure of the various anastomoses. Acute vascular compromise of the loop is usually due to thrombosis of the conduit or the development of a hematoma in the mesentery that can compromise loop integrity. The findings of a black stoma and abdominal pain strongly suggest the development of stomal and possibly loop necrosis (the latter is usually associated with a coexisting urine leak). This problem should be immediately evaluated by loop endoscopy to determine the extent of ischemia. The extent of the bowel ischemia determines the surgical approach used for repair. Dahl DM, McDougal WS: Use of intestinal segments in urinary diversion, Wein, AJ, Kavoussi LR, Novick AC, Partin AW, Peters CA (eds): CAMPBELL-WALSH UROLOGY, ed 10. Philadelphia, Elsevier Saunders, 2012, vol 3, chap 85, p 2436.Adams MC, Joseph DB: Urinary tract reconstruction in children, Wein, AJ, Kavoussi LR, Novick AC, Partin AW, Peters CA (eds): CAMPBELL-WALSH UROLOGY, ed 10. Philadelphia, Elsevier Saunders, 2012, vol 4, chap 129, pp 3484-3485. Question #50 ANSWER=D ______________________________________________________________________________ The KUB reveals speckled intraabdominal calcifications within the abdomen and patent processus vaginalis. The differential diagnosis of this finding includes meconium peritonitis or intrauterine infections of the fetus with toxoplasmosis or cytomegalic virus. Meconium peritonitis occurs when the thickened meconium, caused by cystic fibrosis, plugs the intestinal lumen resulting in obstruction and rupture of the bowel. The in utero bowel perforation heals spontaneously in greater than two-thirds of cases, and extravasated sterile meconium is scattered throughout the abdominal cavity. The meconium is a freely floating peritoneal contaminant, and may move into the scrotum through a patent processus vaginalis resulting in a neonatal hemiscrotal mass. At times, the scrotum may become so full of the meconium that it ruptures (scrotoschisis). If the infant can feed normally and the diagnosis is made, no further surgical intervention is required or needed, as the masses will spontaneously resorb with time. If, however, the bowel perforation persists or the meconium forms a complex intraabdominal mass, it may result in bowel obstruction, both requiring surgery in the neonatal time period. The dense inflammatory reaction that surrounds the meconium within the abdomen makes surgery on these infants extremely problematic. Surgical attempts to excise the mass in the neonate are fraught with hazards and may result in loss of vital organs or severe hemorrhage. Temporary diversion of the gastrointestinal tract above the site of perforation/obstruction and waiting for the inflammation to resolve over several months is recommended prior to definitive surgical repair. Meconium peritonitis is associated with cystic fibrosis in over 50% of the cases. It is therefore recommended that all children with meconium peritonitis should be screened for cystic fibrosis. AFP is elevated in this child due to routine elevations found in neonates. Isolated speckled calcification in the region of the adrenal would be suspicious for adrenal hemorrhage or neuroblastoma. N-myc oncogene is useful for neuroblastoma staging. Chan KL, Tang MH, Tse HY, et al: Meconium peritonitis: Prenatal diagnosis, postnatal management and outcome. PRENATAL DIAGNOSIS 2005;25(8):676-682. Palmer JS: Abnormalities of the external genitalia in boys, Wein, AJ, Kavoussi LR, Novick AC, Partin AW, Peters CA (eds): CAMPBELL-WALSH UROLOGY, ed 10. Philadelphia, Elsevier Saunders, 2012, vol 4, chap 131, p 3555. Question #51 ANSWER=E

Page 26: Live Course Handout.cfm Second Part

______________________________________________________________________________ Spontaneous rupture of a Wilms' tumor immediately results in stage 3 classification, despite the tumor being confined to the kidney. Gross spillage of the tumor has occurred in the flank, and a peritoneal lavage if performed will reveal tumor cells within the abdominal cavity. Appropriate treatment is combined chemotherapy and radiation therapy. In Stage 1, the tumor is confined to the tumor without residual or spillage. In Stage 2, there is extracapsular penetration, and there may be renal sinus extension or tumor thrombus in extrarenal vessels. Stage 3 includes positive abdominal lymph nodes and/or tumor rupture. Ritchey ML, Shamberger RC: Pediatric urologic oncology, Wein, AJ, Kavoussi LR, Novick AC, Partin AW, Peters CA (eds): CAMPBELL-WALSH UROLOGY, ed 10. Philadelphia, Elsevier Saunders, 2012, vol 4, chap 137, pp 3716-3718. Question #52 ANSWER=A ______________________________________________________________________________ A vesicostomy provides optimal upper tract drainage by creating low storage pressures within the bladder. The hydronephrosis and reflux in this child was likely caused by elevated bladder pressures due to a poorly compliant detrusor prior to the vesicostomy. Minimal residual urine in a bladder with a vesicostomy is routinely found when the patient is catheterized in a supine position and is of no consequence without elevated pressure. Stomal revision is not indicated for stenosis since catheterization of vesicostomy is performed with ease. In the absence of stomal stenosis, it is unlikely that an indwelling bladder catheter or a revision of his vesicostomy will affect his UTIs. This patient should undergo assessment for renal nonfunction by a DMSA scan. If a nonfunctional kidney is found, removal of the nonfunctional hydronephrotic system may be of benefit. The best treatment at this age, however, would be to first change antibiotic prophylaxis and then observe for continued or recurrent UTIs. Casale AJ: Posterior urethral valves, Wein, AJ, Kavoussi LR, Novick AC, Partin AW, Peters CA (eds): CAMPBELL-WALSH UROLOGY, ed 10. Philadelphia, Elsevier Saunders, 2012, vol 4, chap 126, pp 3402-3403. Question #53 ANSWER=E ______________________________________________________________________________ Following birth, regulation of renal function is shifted from the placenta to the neonatal kidney. Compared to an adult, the fetal glomerular filtration rate (GFR) is one-tenth of an adult's GFR and doubles by two weeks of age. This increase in function is significantly delayed in infants born prior to 34 weeks of gestation, and in low birth weight infants (<2.5 kg), maturation of neonatal GFR is directly related to an increase in renal blood flow (RBF). This increase in RBF is due to a progressive decrease in renal vascular resistance. The decrease in renal vascular resistance occurs in a step wise fashion preceding from the cortical superficial glomeruli to the intramedullary glomeruli. An infant's GFR

Page 27: Live Course Handout.cfm Second Part

reaches adult levels and coincides with the glomeruli of both cortical and medullary regions being equally perfused at two years of age. Shoskes DA, McMahon AW: Renal physiology and pathophysiology, Wein, AJ, Kavoussi LR, Novick AC, Partin AW, Peters CA (eds): CAMPBELL-WALSH UROLOGY, ed 10. Philadelphia, Elsevier Saunders, 2012, vol 2, chap 38, pp 1025-1028.Arant BS Jr: Fetal and neonatal renal function, in Docimo SG, Canning DA, Khoury AE (eds): CLIN PED UROL, ed 5. London, InForma Healthcare, 2007, chap 21, p 315. Question #54 ANSWER=A ______________________________________________________________________________ The ultrasound demonstrates a classic horseshoe kidney, best noted because of the renal parenchyma (isthmus) over the spine. It represents the most common renal fusion anomaly. It is frequently found in association with other congenital anomalies, including skeletal, cardiovascular, CNS, and anorectal malformations. Horseshoe kidney is seen in 20% of patients with Trisomy 18, and in as many as 60% with Turner's syndrome. If hydronephrosis was present on the ultrasound or the patient is symptomatic, additional studies such as a VCUG, diuretic renogram and possibly a retrograde pyelogram would be indicated. In children with a horseshoe kidney and hydronephrosis, a MAG 3 diuretic renogram will demonstrate a normal washout curve pattern in slightly greater than 80% of cases. Although a multitude of studies have demonstrated that there is an association of horseshoe kidneys with vesicoureteral reflux with grade 2 or higher VUR found in up to 30% of the affected patients. The presence of asymptomatic reflux is currently believed to be of no clinical significance. Therefore, recommendations to obtain a VCUG is limited to patients found to have ultrasound findings of a dilated ureter, hydronephrosis, or a history of UTIs. Shapiro E, Bauer SB, Chow JS: Anomalies of the upper urinary tract, Wein, AJ, Kavoussi LR, Novick AC, Partin AW, Peters CA (eds): CAMPBELL-WALSH UROLOGY, ed 10. Philadelphia, Elsevier Saunders, 2012, vol 4, chap 117, pp 3145-3149. Question #55 ANSWER=A ______________________________________________________________________________ The superior mesenteric artery gives rise to the right colic and ileocolic artery. The ileocolic artery supplies the terminal ileum and cecum along with the appendix. The ileocolic artery is the beginning of the arc of Drummond, a vascularly rich anastomotic blood supply to the ileum, colon, and rectum. The arc of Drummond is fed by the ileocolic, right colic, middle colic, left colic, sigmoid, and hemorrhoidal arteries. Knowledge of the blood supply to the appendix, terminal ileum, and colon and how that may have been impacted by prior surgical procedures is critical in the preoperative assessment of the patient that is contemplating major genitourinary reconstructive surgery. Dahl DM, McDougal WS: Use of intestinal segments in urinary diversion, Wein, AJ, Kavoussi LR, Novick AC, Partin AW, Peters CA (eds): CAMPBELL-WALSH UROLOGY, ed 10. Philadelphia, Elsevier Saunders, 2012, vol 3, chap 85, pp 2413-2414.

Page 28: Live Course Handout.cfm Second Part

Question #56 ANSWER=E ______________________________________________________________________________ Patients with increased intraabdominal pressure (such as those on peritoneal dialysis or those with a ventriculoperitoneal shunt) should have the contralateral groin explored at the time of an ipsilateral communicating hydrocele repair, as they have been shown to have a high rate of metachronous contralateral inguinal hernia or communicating hydrocele. Otherwise, no consensus exists regarding exploration of the clinically normal side, although one may find a contralateral patent process in up to 60% of explored contralateral asymptomatic sides. In studies that have noted the appearance of a patent processus vaginalis on the contralateral asymptomatic side at laparoscopy, the development of symptoms related to the patent processus vaginalis occurs in <10%, with almost all symptomatic patients having coexisting pathology that results in elevated intraabdominal pressures and/or a prior history of symptomatic hydrocele/hernia in early infancy. In children with high anesthetic risk, some surgeons recommend prophylactic contralateral exploration in individuals with a unilateral communicating hydrocele to avoid an additional anesthesia. Barthold JS: Abnormalities of the testes and scrotum and their surgical management, Wein, AJ, Kavoussi LR, Novick AC, Partin AW, Peters CA (eds): CAMPBELL-WALSH UROLOGY, ed 10. Philadelphia, Elsevier Saunders, 2012, vol 4, chap 132, p 3585. Question #57 ANSWER=E ______________________________________________________________________________ When identifying the ureter outside the bladder, it is helpful to understand the relationship between the ureter, bladder, and the obliterated umbilical artery. The ureter lies posteromedial to the obliterated umbilical artery. Once the obliterated umbilical artery is identified, it can be ligated, thus gaining direct access to the ureter that lies lateral to the bladder wall. Knowledge of the relationship between the obliterate umbilical artery, ureter, and bladder is extremely helpful for any extravesical approach to the ureter. Anderson JK, Cadeddu JA: Surgical anatomy of the retroperitoneum, adrenals, kidneys, and ureters, Wein, AJ, Kavoussi LR, Novick AC, Partin AW, Peters CA (eds): CAMPBELL-WALSH UROLOGY, ed 10. Philadelphia, Elsevier Saunders, 2012, vol 1, chap 1, pp 28-29. Question #58 ANSWER=C ______________________________________________________________________________ The femoral nerve arises from L2-L4, runs through the psoas muscle, and emerges at its lower lateral border, splitting into an anterior and posterior branch at the level of the inguinal ligament. Its motor branches serve to extend the knee, and its sensory branches are responsible for sensation over the anterior and medial aspects of the thigh, medial shin, and arch of the foot. The femoral nerve is the most common nerve injured in a psoas hitch procedure. Injury usually occurs due to neural entrapment by

Page 29: Live Course Handout.cfm Second Part

sutures placed deep into the psoas muscle. To prevent this injury, it is recommended to place the sutures moderately superficial and parallel to the psoas muscle fibers. The obturator nerve arises from L2-L4 and also descends through the fibers of the psoas muscle on its medial border near the brim of the pelvis. It then passes behind the common iliac arteries on the lateral side of the internal iliac artery and ureter, and runs along the lateral wall of the pelvis, exiting at obturator foramen. It provides motor supply to the adductors of the thigh and sensation to the medial thigh, and is rarely injured during a psoas hitch procedure. The neural injury described in this patient comes from damage to the genitofemoral nerve. The genitofemoral nerve arises from the L1-L2 segments, and is the more caudal of the two nerves that are easily seen on the anterior surface of the psoas major muscle. The nerve divides into two branches, the genital branch and the femoral branch. The genital branch passes through the inguinal canal, where it is frequently seen during a inguinal dissection. In men, the genital branch supplies sensation to the mid- and lower scrotal skin. In women, the genital branch accompanies the round ligament of the uterus, supplying sensation to the middle and lower areas of the mons pubis and labia majora. The femoral branch of the genitofemoral nerve serves as the motor supply to the cremasteric muscle, and has a sensory function to the anterior upper thigh. The genitofemoral nerve is easily visualized during the dissection for performing a psoas hitch; it is rarely injured during the dissection due to this ease in visibility and its tendency to be displaced away from the sutures. The ilioinguinal nerve arises from L1 and runs on the lateral border of the psoas major. It is always cephalad to the genitofemoral nerve, and is easily seen on the anterior surface of the psoas muscle. It travels obliquely across the quadratus lumborum and iliacus muscles, perforating the transversus abdominis near the anterior part of the iliac crest. From there, it pierces the internal oblique muscle and distributes motor fibers to it, then accompanies the spermatic cord through the external inguinal ring. In the male, it provides sensation to the skin over the base of the penis and upper part of the scrotum. In the female, it provides sensation to the base of the clitoris and upper portion of the mons pubis and labium majorum. Similar to the genitofemoral nerve, it is easily visible at the time of performing a psoas hitch, and its visibility will usually prevent its injury during the operation. The sacral nerves arise from S1-S5, and each nerve emerges in two divisions: one division through the anterior sacral foramina and the other division through the posterior sacral foramina. The five neural roots exchange multiple neural fibers, and exist as a neural plexus that have both motor and sensory afferent and efferent fibers. They are responsible for part of the sensory perception and the movements of the lower extremities, and also contain parasympathetic fibers that supply the descending colon and rectum, urinary bladder, and genital organs. Chung BI, Sommer G, Brooks JD: Anatomy of the lower urinary tract and male genitalia, Wein, AJ, Kavoussi LR, Novick AC, Partin AW, Peters CA (eds): CAMPBELL-WALSH UROLOGY, ed 10. Philadelphia, Elsevier Saunders, 2012, vol 1, chap 2, p 47. Question #59 ANSWER=A ______________________________________________________________________________ During an orchidopexy, once the processus vaginalis has been ligated proximal to the internal ring and a retroperitoneal dissection performed, it is not generally advisable to divide the spermatic vessels (Fowler-Stevens maneuver) to place the testis in the scrotum. It is hypothesized that the existing surgical dissection has impaired the collateral blood supply between the distal spermatic vessels and the vas. This vascular compromise occurs during the dissection of the processus vaginalis off of the terminal aspect of the cord. Options in this scenario are to fix the testis to the pubic symphysis and plan for a second stage

Page 30: Live Course Handout.cfm Second Part

at a later date; however, the secondary dissection is tedious and associated with significantly increased complication rate. Placing the testis in the scrotum using tension suture fixation. This procedure has a high incidence of reported failure with either testicular atrophy ensuing and/or retraction of the testis back into the superficial inguinal pouch. The optimal management in this scenario is with division of the inferior epigastric vessels and movement of the spermatic cord to a more medial path, the Prentiss maneuver. MRI evaluations have revealed that division of the inferior epigastric vessels and repositioning of the spermatic cord medially will routinely decrease the length the testis must travel to reach the scrotum by approximately 1.5 cm. Schneck FX, Bellinger MF: Abnormalities of the testes and scrotum and their surgical management, in Wein AJ, Kavoussi LR, Novick AC, Partin AW, Peters CA (eds): CAMPBELL'S UROLOGY, ed 9. Philadelphia, Saunders Elsevier, 2007, vol 4, chap 127, p 3779.Prakasha J, Dalelaa D, Goela A, Dalelaa D, Kumara M, Sankhwara S, Kureelb S: Testicular rerouting by modified Prentiss maneuver: Usefulness in bilateral synchronous orchidopexy for high inguinal undescended testes. J PED SUR 2014;49 1311-1314. Question #60 ANSWER=A ______________________________________________________________________________ In the presence of a 46 XY karyotype, perineal hypospadias and palpable gonads, the most likely diagnosis is 5-alpha-reductase deficiency. The diagnosis is implied by demonstrating a high ratio of testosterone to dihydrotestosterone, revealing the lack of the converting enzyme 5-alpha-reductase. It is confirmed by documenting an abnormality within the 5-alpha-reductase genome. Plasma progesterone and plasma 17-hydroxyprogesterone are routinely obtained to diagnosis 21-hydroxylase deficiency, the most common form of congenital adrenal hyperplasia. Abnormalities in 21-hydroxylase is associated with masculinization of a female genotype (46 XX), but is not associated with feminization of a male genotype (46 XY). Mullerian inhibitory factor levels are obtained in the newborn to assess for the presence of testes and/or when persistent Mullerian structures are noted during the evaluation. An abdominal pelvic ultrasound is useful in evaluating disorders of sexual differentiation, especially to evaluate for the presence of gonads (palpable in this case), or when searching for Mullerian remnants. Although a VCUG could be obtained to exclude the presence of an enlarged prostatic utricle, it would not aid in the diagnosis of the underlying etiology of the phenotypic abnormality. Diamond DA, Yu RN: Sexual differentiation: Normal and abnormal, Wein, AJ, Kavoussi LR, Novick AC, Partin AW, Peters CA (eds): CAMPBELL-WALSH UROLOGY, ed 10. Philadelphia, Elsevier Saunders, 2012, vol 4, chap 133, pp 3623-3624. Question #61 ANSWER=D ______________________________________________________________________________ Bladder stones are not infrequent in patients who have undergone bladder augmentation. The most important adjunct for stone prevention is consistent CIC and bladder irrigation. The irrigation is important in clearing thicker mucus, which may form clumps in the bladder and be a major initiator of stone formation. UTI may also occur in these patients, but they are thought to be the result of stone

Page 31: Live Course Handout.cfm Second Part

formation not thought to be causative in most cases. For the augmented patient who turns out to be a chronic stone former, the use of a mucolytic agent can be a useful second tier adjunctive measure. Both dilute urea solutions and acetylcysteine have been used for this purpose, and are instilled once daily (or periodically) via the catheter during the CIC process. Patients who form stones after augmentation, but who did not have previous issues and have no upper tract stones are unlikely to have a metabolic predisposition to stone formation. Therefore, in the absence of associated nephrolithiasis, the 24-hour urine assessment is unlikely to be of clinical benefit. Routine alkalinization of the urine with Polycitra-K is not helpful because the urine is almost inevitably alkaline due to the normal bowel secretion. In this patient population, the mucus is the principal problem that needs to be addressed. Adams MC, Joseph DB: Urinary tract reconstruction in children, Wein, AJ, Kavoussi LR, Novick AC, Partin AW, Peters CA (eds): CAMPBELL-WALSH UROLOGY, ed 10. Philadelphia, Elsevier Saunders, 2012, vol 4, chap 129, pp 3483-3484. Question #62 ANSWER=A ______________________________________________________________________________ Bladder emptying is characterized by activation of the pelvic nerve (parasympathetics S2-S4) and quiescence of the hypogastric (sympathetics T-10-L-2) and pudendal nerves (somatic S2-S4), the latter of which will result in relaxation of the external sphincter. Bladder storage would be characterized by the opposite activity of each of these three neural influences. Yeung CK, Sihoe JDY: Non-neuropathic dysfunction of the lower urinary tract in children, Wein, AJ, Kavoussi LR, Novick AC, Partin AW, Peters CA (eds): CAMPBELL-WALSH UROLOGY, ed 10. Philadelphia, Elsevier Saunders, 2012, vol 4, chap 127, pp 3412-3413. Question #63 ANSWER=A ______________________________________________________________________________ Based on the intraoperative photograph, the working diagnosis is persistent Mullerian duct syndrome versus ovotestes syndrome. Both gonads are visualized as well as the uterus and Fallopian tubes (Mullerian remnant). The finding that both gonads also have areas of cyst formation is concerning for a possible ovotesticular disorder. The next step is to biopsy both gonads, replace them within the abdomen, halt the procedure, and then perform karyotype and genetic evaluation for the Mullerian inhibitor gene and receptor. Definitive surgical correction should not be performed until the correct diagnosis is available and the family appropriated advised. Diamond DA, Yu RN: Sexual differentiation: Normal and abnormal, Wein, AJ, Kavoussi LR, Novick AC, Partin AW, Peters CA (eds): CAMPBELL-WALSH UROLOGY, ed 10. Philadelphia, Elsevier Saunders, 2012, vol 4, chap 133, pp 3624-3625. Question #64 ANSWER=A ______________________________________________________________________________

Page 32: Live Course Handout.cfm Second Part

Studies by Yeung and associates have extensively examined the lower urinary tract in children with isolated nocturnal enuresis. In children failing desmopressin, the majority of patients will show markedly reduced functional bladder capacity or detrusor overactivity. Due to these findings, it is recommended that an empiric trial of antimuscarinics at bedtime should be considered in patients who fail desmopressin. Decreased bladder compliance, increased bladder capacity for age, and elevated urinary residuals are not routinely seen in this patient's with primary nocturnal enuresis. A prolonged EMG lag time is a lengthening of the time between the start of relaxation of the external urinary sphincter and the start of urine flow. A prolonged EMG lag time is one of the chief urodynamic findings in primary bladder neck dysfunction (PBND), and can be used in place of a more invasive videourodynamic study to make this diagnosis. Individuals with prolonged EMG response time will frequently respond to treatment with alpha-blockers. Response to successful treatment can be monitored by a decrease in the EMG lag interval. This abnormality is not associated with primary nocturnal enuresis. Yeung CK, Sihoe JDY: Non-neuropathic dysfunction of the lower urinary tract in children, Wein, AJ, Kavoussi LR, Novick AC, Partin AW, Peters CA (eds): CAMPBELL-WALSH UROLOGY, ed 10. Philadelphia, Elsevier Saunders, 2012, vol 4, chap 127, pp 3427-3430. Question #65 ANSWER=D ______________________________________________________________________________ Patients undergoing enterocystoplasty with either small or large bowel will absorb chloride from urine within the augmented bladder. This will lead to the simultaneous absorption of hydrogen ions and resultant potential acidosis. In patients with normal renal function, this is generally well-tolerated, as the kidneys can maintain homeostasis. In patients with underlying renal damage, however, this can become problematic. This patient has a history of PUV associated with a small kidney, and raises the concern of diminished renal function. In this circumstance, it is very helpful to assess the acid-base status of the patient before potentially worsening this with an enterocystoplasty. A low serum bicarbonate level would indicate baseline acidosis from renal insufficiency, and is valuable in this regard. Beyond this finding, it should be noted that patients with a history of substantial urinary obstruction, even once relieved, may have a prominent and persistent concentrating defect, nephrogenic diabetes insipidus, which will result in a high obligate daily urine output. This should be recognized before the procedure so that the surgeon may choose an appropriate length of bowel segment to create adequate bladder capacity to handle the urine volume that will be present post-op. Adams MC, Joseph DB: Urinary tract reconstruction in children, Wein, AJ, Kavoussi LR, Novick AC, Partin AW, Peters CA (eds): CAMPBELL-WALSH UROLOGY, ed 10. Philadelphia, Elsevier Saunders, 2012, vol 4, chap 129, pp 3481-3482. Question #66 ANSWER=B ______________________________________________________________________________

Page 33: Live Course Handout.cfm Second Part

During surgery, if the ureter has been noted to be clipped or ligated, the clip or ligature should be removed. The ureter should be assessed for necrosis or urinary leakage. If one or both of these symptoms are present, a primary end-to-end anastomosis is the preferred management plan for a proximal injury, while a ureteral reimplantation may be considered for an injury to the distal one-third of the ureter. Transureteroureterostomy should not be considered in the acute trauma setting, since the exact extent of the ureteral injury at times is hard to estimate, and this procedure places the contralateral renal unit at jeopardy. If a significant portion of the ureter appears, injured, placement of a nephrostomy with delayed repair should be a consideration. If there is no evidence of necrosis or a urine leak, cystoscopy and placement of a ureteral stent for a four to six week period is preferred. Husmann DA: Pediatric genitourinary trauma, Wein, AJ, Kavoussi LR, Novick AC, Partin AW, Peters CA (eds): CAMPBELL-WALSH UROLOGY, ed 10. Philadelphia, Elsevier Saunders, 2012, vol 4, chap 138, p 3745.Santucci RA, Doumanian LR: Upper urinary tract trauma, Wein, AJ, Kavoussi LR, Novick AC, Partin AW, Peters CA (eds): CAMPBELL-WALSH UROLOGY, ed 10. Philadelphia, Elsevier Saunders, 2012, vol 2, chap 42, p 1186. Question #67 ANSWER=D ______________________________________________________________________________ The ultrasound findings indicate bladder outlet obstruction in this male fetus. Fetal urine is normally hypo-osmolar. This bladder tap demonstrates hyperosmolar urine, and is indicative of poor renal function with impaired reabsorption capacity. It is important to repeat the urine chemistries on sequential taps to assess for improvement in urine chemistries after decompression (as the initial urine sample may represent equilibration due to stasis of the bladder urine). If on subsequent serial bladder chemistries the same findings are demonstrated, this would predict a poor prognosis regarding postnatal renal function and patient survival. In this case, consideration for termination of pregnancy could be addressed. If, however, on repeat bladder tap the urine chemistries are improved to normal range, this would indicate a better prognosis for recoverability and postnatal renal function, which would influence management options toward consideration of in utero intervention. Lee RS, Borer JG: Perinatal urology, Wein, AJ, Kavoussi LR, Novick AC, Partin AW, Peters CA (eds): CAMPBELL-WALSH UROLOGY, ed 10. Philadelphia, Elsevier Saunders, 2012, vol 4, chap 114, pp 3062-3064. Question #68 ANSWER=B ______________________________________________________________________________ Metabolic acidosis is associated with hyperchloremia, hyperkalemia, compensatory increased respiratory rate, increased renal production of ammonia to compensate for the acidosis, and decreased vascular resistance, the latter may lead to under perfusion of the extremities worsening the acidotic load. Sodium concentration is not affected.

Page 34: Live Course Handout.cfm Second Part

Greenbaum LA: Electrolyte disorders, in Kliegman A, Stanton BF, Schor NF, St. Geme, JW III, Behrman RE (eds): NELSON'S TEXTBOOK OF PEDIATRICS, ed 19. Philadelphia, Elsevier Saunders Publishing, 2011, chap 52. Question #69 ANSWER=C ______________________________________________________________________________ This patient has evidence of substantial proteinuria and symptoms consistent with glomerulonephritis. The healing caruncle suggests the potential of a prior streptococcal infection, and now the sequelum of post-strep glomerulonephritis. Post-strep GN patients almost universally have a low C3 level due to consumption of this component of the complement cascade. Those with pharyngeal strep will usually have elevated ASO titers, while those with skin strep will not. Alternatively, those with a skin source of strep A infection will have altered anti-DNAase B titers at the time of glomerulonephritic presentation. This allows the clinician to convincingly conclude the etiology of the GN and avoid renal biopsy, as well as counsel the family and follow the child in an appropriate and supportive manner. Renal ultrasound is indicated in some cases of gross hematuria when stone or mass is on the differential list. Palmer LS, Trachtman H: Renal functional development and diseases in children, Wein AJ, Kavoussi LR, Novick AC, Partin AW, Peters CA (eds): CAMPBELL-WALSH UROLOGY, ed 10. Philadelphia, Elsevier Saunders, 2012, vol 4, chap 112, p 3011.Mehta A, Faizan MK, Caldamone AA: Hematuria: Gross and microscopic, Palmer JS (ed): CURRENT CLINCAL UROLOGY: PEDIATRIC UROLOGY. New York City, Humana Press, 2011, chap 5, pp 99-100. Question #70 ANSWER=A ______________________________________________________________________________ The early diagnosis of renal papillary necrosis is important to improve prognosis and reduce morbidity of this disorder. Since ischemia with metabolic acidosis and hypovolemia are prominent underlying factors in the development of renal papillary necrosis, prompt resuscitation of the patient's fluid volume, treatment of hypoxia, and urinary alkalinization is the foundation of treatment. If the urinalysis suggests bacteriuria, broad-spectrum I.V. antibiotics should be initiated. Glycemic control is critical in patients with diabetes associated induced papillary necrosis. At baseline, children with sickle cell disease are anemic. An exchange transfusion to decrease HbS is almost never necessary unless serial hemoglobins reveal a drop in value. If the urinalysis suggests signs of infection, treatment with antibiotics is mandatory. Most sloughed papillae will spontaneously pass without sequale. If evidence of symptomatic ureteral obstruction is present, especially if there is concurrent pyuria, a ureteral stent should be placed. A CT urogram is not recommended and may actually potentiate the disease process. Specifically, volume depletion is almost universally present at the time of sickle crisis. I.V. contrast administration in the presence of hypovolemia will induce additional papillary necrosis and should be deferred until the patient has undergone I.V. hydration. Schaeffer AJ, Schaeffer EM: Infections of the urinary tract, Wein, AJ, Kavoussi LR, Novick AC, Partin AW, Peters CA (eds): CAMPBELL-WALSH UROLOGY, ed 10. Philadelphia, Elsevier Saunders, 2012, vol 1, chap 10, pp 268-269.

Page 35: Live Course Handout.cfm Second Part

Question #71 ANSWER=D ______________________________________________________________________________ In the presence of a medical history and physical examination suspicious for an ectopic ureter, normal abdominal ultrasound, and VCUG, either a MR urogram or CT scan would be the next appropriate study. These evaluations are more accurate in defining the anatomy of a small non- or poorly functioning ectopic segment. A DMSA scan may or may not show function in a small renal duplication anomaly and can easily miss the diagnosis in small upper pole segments. A cystoscopy would be an alternative; however, it can also be inconclusive in the face of a normal appearing trigone and intravesical ureteral orifices, and locating an ectopic orifice within the vagina is extremely problematic. The upper pole segments are classically poorly secreting and will usually fail to excrete either Pyridium or methylene blue. Timed voiding, maintenance of a voiding calendar, and uroflow/EMG would be alternatives only if an ectopic ureter has been definitely ruled out. Antimuscarinics would be an option if an ectopic ureter has been ruled-out and the patient failed timed voiding. Peters CA, Schlussel RN, Mendelsohn C: Ectopic ureter, ureterocele, and ureteral anomalies, Wein, AJ, Kavoussi LR, Novick AC, Partin AW, Peters CA (eds): CAMPBELL-WALSH UROLOGY, ed 10. Philadelphia, Elsevier Saunders, 2012, vol 4, chap 121, p 3245. Question #72 ANSWER=A ______________________________________________________________________________ The incidence of sibling reflux in an index patient is 25% to 30%. The clinical significance of diagnosing incidental reflux in this population is uncertain and results in the dilemma of whether or not to screen for the presence the asymptomatic reflux. Options of screening advanced by the AUA Guideline Panel on this subject suggest: Option 1: In an asymptomatic sibling, an observational approach without screening for VUR can be followed. If an acute UTI develops, prompt treatment should be instituted, and subsequent evaluation by a renal ultrasound and VCUG obtained. Option 2: In an asymptomatic sibling, a renal and bladder ultrasound may be performed to identify significant renal scarring or hydronephrosis. If an abnormality is found, a VCUG should be obtained. Option 3: In siblings with a history of UTIs, a VCUG should be obtained. It should be noted that a renal ultrasound will not rule-out reflux. If there is a structural abnormality of the kidney or evidence of ureteral dilation, then it is quite likely that significant reflux (Grade 3 or higher) would be disclosed if a VCUG is performed. In essence, it is recommended that a VCUG or nuclear cystogram is recommended to be reserved for patients with evidence of an abnormality on renal ultrasound or in siblings with a history of UTIs. Peters CA, Skoog SJ, Arant BS Jr, et al: Management and screening of primary vesicoureteral reflux in children: AUA GUIDELINE. Revised, 2010. American Urological Association Education and Research, Inc, 2010. Topic #4 Screening of siblings and offspring of patients with vesicoureteral reflux.http://www.auanet.org/education/guidelines/vesicoureteral-reflux-a.cfm

Page 36: Live Course Handout.cfm Second Part

Question #73 ANSWER=A ______________________________________________________________________________ The ultrasound demonstrates a cystic mass with internal echoes, indicating debris at the base of the penis in the pendulous area of the urethra. The retrograde urethrogram shows an external impression on the mid-bulbar region of the urethra. These findings are consistent with a cyst of Cowper’s duct which has become enlarged and possibly infected. Cowper’s glands are located proximal to the external sphincter, but the duct drains into the mid-bulbar region of the urethra. They may be found incidentally on cystoscopy where a blue dome cystic lesion is seen on the floor of the mid-bulbar urethra. A urethral duplication would have a separate urethral channel. A urethral diverticulum would have an outpouching off of the urethra which may cause obstruction of the urethra due to a distal common lip. Glands of Littré are located in the penile urethra. Bogaert GA: Urethral duplication and other urethral anomalies, in Gearhart JP, Rink RC, Mouriquand PDE (eds): PEDIATRIC UROLOGY, ed 2. Philadelphia, Saunders/Elsevier 2010, chap 34, p 454. Question #74 ANSWER=E ______________________________________________________________________________ Both bladder neck lacerations and urethral injuries are known to be associated with traumatic pelvic fractures, with prepubertal children at an approximately two-fold higher risk for their occurrence than adults. In the presence of a bladder neck injury, there is a substantial risk that the urine will continue to extravasate into the pelvis despite placement of a suprapubic tube. Continued urinary extravasation results in an increased occurrence of pelvic hematoma infection, pelvic abscess, and pubic osteomyelitis. When a bladder neck injury is suspected, it is appropriate to explore and repair the bladder neck laceration to avoid further complications. Opening the hematoma extravesically will increase the risk of intraoperative bleeding and post-op infection. It is better to explore by opening the bladder toward the dome and visualizing and repairing the injury from an intravesical location when possible. Husmann DA: Pediatric genitourinary trauma, Wein, AJ, Kavoussi LR, Novick AC, Partin AW, Peters CA (eds): CAMPBELL-WALSH UROLOGY, ed 10. Philadelphia, Elsevier Saunders, 2012, vol 4, chap 138, pp 3746-3747. Question #75 ANSWER=B ______________________________________________________________________________ Autonomic dysreflexia describes an abrupt outpouring of sympathetic nervous discharge that results in hypertension, headache, reflex bradycardia, and diaphoresis above the level of the spinal cord lesion. This most commonly occurs in patients with complete cord lesions above the sympathetic outflow tracts located at T-10 to L-2. The risk of autonomic dysreflexia occurs in complete spinal cord lesions above T-8. Patients must have an intact distal spinal cord below the lesion. An intact distal cord will result in the stimulation of sensory afferent bladder fibers by bladder distension. To accommodate for the stretching phenomena stimulated by the bladder, an increase in sympathetic tone occurs to allow for accommodation of urine in the bladder. Due to the complete cord lesion above the level of sympathetic

Page 37: Live Course Handout.cfm Second Part

outflow (T10-L2), the sympathetic response can no longer be modulated by the pontine micturition center. This will result in an unbalanced sympathetic neural stimulation with excessive catecholamines release, resulting in hypertension, reflex bradycardia, and diaphoresis above the level of the lesion. The resultant unregulated discharge can cause seizures, retinal hemorrhage, pulmonary edema, myocardial infarction, cerebral hemorrhage, and death. Treatment of autonomic dysreflexia is by emptying the bladder, loosening all clothing and verifying the absence of fecal impaction. If symptoms persist, 1/2 to 1 inch of nitropaste applied above the level of the lesion is the preferred method of treatment. This allows the physician to treat for rebound hypotension, if it occurs, by removal of the paste. Nifidipine, once the drug of choice for treatment has been found to be associated with rebound hypotension, subsequent stroke and death and is no longer recommended for use in patients with autonomic dysreflexia. The physician should be aware that PDE5 inhibitors used in conjunction with nitroglycerin may cause profound rebound hypotension. In spinal cord injured patients routinely taking PDE5 inhibitors for treatment of their erectile dysfunction, captopril 25 mg sublingually, is currently the drug of choice. In a patient with a history of autonomic dysreflexia pretreatment by captopril or alpha blockers 30-60 min prior to any invasive urologic procedure i.e., urodynamic studies, cystoscopy or electroejaculation, will aid in its prevention. MacLellan DL, Bauer SB: Neuropathic dysfunction of the lower urinary tract, Wein, AJ, Kavoussi LR, Novick AC, Partin AW, Peters CA (eds): CAMPBELL-WALSH UROLOGY, ed 10. Philadelphia, Elsevier Saunders, 2012, vol 4, chap 128, p 3455. Question #76 ANSWER=B ______________________________________________________________________________ The radiographic images of this patient show a 1 cm lower pole renal laceration with a subcapsular hematoma, a grade 2 renal injury. Recommendations for the follow-up of a traumatic renal injury is based on the grade of the injury. Individuals sustaining a grade 1 injury should undergo a urinalysis and blood pressure evaluation one-week post-injury. If normal, they may resume all activities. Individuals with a grade 2-3 injury (with all fragments viable) should be restricted from athletic activities and strenuous work for six weeks. A urinalysis is performed, and blood pressure is checked at six weeks. If these values are normal, the patient may resume activities, and no additional radiographic assessments are necessary. By a consensus statement from the AUA Guideline Panel, individuals with a grade 3 renal injury (with devitalized renal fragments) and grade 4 and 5 renal injuries should be restricted from athletic events and strenuous work for six weeks. At six weeks, the patient should be evaluated with a BP evaluation, urinalysis, and reimaging with either ultrasound or CT scan as an option at that time. If any patient has persistent hematuria or elevation of BP at six weeks, reimaging and consideration for cystoscopy to determine a source for the hematuria is strongly recommended. There is no need to reimage or assess individual prior to six weeks of follow-up unless warranted by changing clinical symptoms. Shoskes DA, McMahon AW: Renal physiology and pathophysiology, Wein, AJ, Kavoussi LR, Novick AC, Partin AW, Peters CA (eds): CAMPBELL-WALSH UROLOGY, ed 10. Philadelphia, Elsevier Saunders, 2012, vol 2, chap 38, pp 1082-1083.Morey, A: AUA guideline panel on trauma http:// www.auanet.org/education/guidelines/urotrauma.cfm.Breen KJ: Blunt renal trauma: Routine follow-up imaging is excessive. UROL 2014;84:62-67.

Page 38: Live Course Handout.cfm Second Part

Question #77 ANSWER=A ______________________________________________________________________________ The use of screening urinalysis has been practiced in the past in the USA and other countries with the finding of microscopic hematuria (>3 RBC/hpf) found in up to three percent of patients. This results in a large number of follow-up visits and exams with an extremely low yield in terms of clinically significant pathology. For this reason, the AAP Guidelines now recommend no routine (screening) urinalyses in asymptomatic children of any age. Palmer LS, Trachtman H: Renal functional development and diseases in children, Wein, AJ, Kavoussi LR, Novick AC, Partin AW, Peters CA (eds): CAMPBELL-WALSH UROLOGY, ed 10. Philadelphia, Elsevier Saunders, 2012, vol 4, chap 112, p 3004. Question #78 ANSWER=B ______________________________________________________________________________ According to the 2014 AUA Guideline on cryptorchidism, at the time of exploration for a non-palpable testis in boys, surgical specialists should identify the status of the testicular vessels to help determine the next course of action. Testicular vessels with a blind ending are indicative of testicular absence. Identification of the vas deferens does not preclude further search for the testicular artery. Contralateral testicular hypertrophy is not 100% sensitivity for testicular absence. Neither a patent processes vaginalis nor a Mullerian remnant indicates testicular absence. Evaluation and treatment of cryptorchidism: AUA Guideline. Kolon TF, Herndon CDA, et al. J UROL 2014;192:337-345.Kolon TF, Herndon CDA, Baker LA, et al: Evaluation and treatment of cryptorchidism: AUA GUIDELINE. American Urological Association Education and Research, Inc, 2014.http://www.auanet.org/education/guidelines/cryptorchidism.cfm Question #79 ANSWER=C ______________________________________________________________________________ In spite of successful treatment of this patient's posterior urethral valves, he documents progressive renal failure. This is probably due to his underlying congenital renal dysplasia, but it may be aggravated by the recurrent episodes of pyelonephritis. Optimal management of this patient is with a vesicostomy. This procedure will remove any possible bladder component (high storage pressures) that could be affecting renal function and provide optimal bladder drainage to decrease the risk of renal damage from further infections. Although treatment with Deflux or ureteral reimplantation are options, they would not remove any possible bladder component that may be impairing renal function. This patient is already on maximal doses of antimuscarinics for age; increasing the dose further would result in increased side effects, and would be of little benefit in preventing the recurrent febrile UTIs. Overnight drainage would do little to protect the kidneys from infection, and would be socially problematic in this patient that had previously failed a trial of CIC.

Page 39: Live Course Handout.cfm Second Part

Casale AJ: Posterior urethral valves, Wein, AJ, Kavoussi LR, Novick AC, Partin AW, Peters CA (eds): CAMPBELL-WALSH UROLOGY, ed 10. Philadelphia, Elsevier Saunders, 2012, vol 4, chap 126, pp 3398-3403. Question #80 ANSWER=D ______________________________________________________________________________ The image shows a hypoechoic perinephric fluid collection consistent with fluid, most likely urine. Perinephric urinomas and urinary ascites may occur in the presence of bladder outlet obstruction from posterior urethral valves. The high intraluminal pressures cause urinary extravasation by rupturing through a renal fornix. Urine will then enter into the retroperitoneum. Ascites occur when the fluid crosses the peritoneum as a transudate. The forniceal rupture may be protective of renal function. In this case, the bilateral pneumothoraces are indicative of high respiratory pressures need to provide adequate oxygenation in the presence of pulmonary hypoplasia. Lee RS, Borer JG: Perinatal urology, Wein, AJ, Kavoussi LR, Novick AC, Partin AW, Peters CA (eds): CAMPBELL-WALSH UROLOGY, ed 10. Philadelphia, Elsevier Saunders, 2012, vol 4, chap 114, pp 3049-3050.Casale AJ: Posterior urethral valves, Wein, AJ, Kavoussi LR, Novick AC, Partin AW, Peters CA (eds): CAMPBELL-WALSH UROLOGY, ed 10. Philadelphia, Elsevier Saunders, 2012, vol 4, chap 126, pp 3393-3394. Question #81 ANSWER=C ______________________________________________________________________________ In a patient with a history of lipomeningocele complaining of new onset of urinary incontinence, four major etiologies for worsening incontinence need to be entertained. They are: 1) UTI, 2) constipation, 3) age inappropriate antimuscarinics, and 4) development of spinal cord tethering or spinal syrinx. In this clinical scenario, patients are usually given an empiric trial of antibiotic therapy or a urine culture obtained to rule-out the presence of an infection. Management of the GI tract should be verified and an increased dose of antimuscarinics prescribed if an age inappropriate level were previously being administered. If symptoms persist, urodynamic studies should be performed. Alterations in the current studies from prior baseline urodynamic evaluations is highly concerning for possible development of a tethered spinal cord or development of spinal syrinx, and should prompt an MRI scan evaluation of the spine and neurosurgical consultation. There is no indication for bladder augmentation or onabotulinumtoxinA injection at this point. Likewise, increasing CIC frequency alone is not helpful, as any underlying structural cord abnormality needs to be addressed. MacLellan DL, Bauer SB: Neuropathic dysfunction of the lower urinary tract, Wein, AJ, Kavoussi LR, Novick AC, Partin AW, Peters CA (eds): CAMPBELL-WALSH UROLOGY, ed 10. Philadelphia, Elsevier Saunders, 2012, vol 4, chap 128, pp 3446-3447, 3452. Question #82 ANSWER=C

Page 40: Live Course Handout.cfm Second Part

______________________________________________________________________________ Twenty-four hour urinary studies reveal low urinary calcium excretion and high urinary oxalate levels. This physiologic finding is consistent with inadequate oral calcium intake resulting in unbound intestinal oxalate. The unbound intestinal oxalate results in hyperabsorption of oxalate from the GI tract and excess oxalate excreted in the urine. The first two steps in treatment should be to increase urine output to >2000 ml per day and increase dietary calcium. The increase in calcium in the gut will bind to the oxalate and decrease oxalate absorption. Normal serum calcium and phosphate with low urinary calcium is not consistent with hyperparathyroidism and checking serum parathormone is therefore not necessary. A thiazide diuretic may decrease urinary calcium excretion, but that will not address the low gut calcium or decrease urinary oxalate, and will have minimal effect on recurrent calcium oxalate stone formation. Potassium citrate or other forms of urinary alkalinization (e.g. orange juice or lemonade) may be a useful adjunct if stones form despite dietary modification and increased fluid intake. Ferrandino MN, Pietrow PK, Preminger GM : Evaluation and medical management of urinary lithiasis, Wein, AJ, Kavoussi LR, Novick AC, Partin AW, Peters CA (eds): CAMPBELL-WALSH UROLOGY, ed 10. Philadelphia, Elsevier Saunders, 2012, vol 2, chap 46, pp 1297, 1308.Pearle MS, Goldfarb DS, Assimos DG, et al: Medical management of kidney stones: AUA Guideline. American Urological Association Education and Research, Inc, 2014.http://www.auanet.org/education/guidelines/management-kidney-stones.cfm Question #83 ANSWER=D ______________________________________________________________________________ The patient has marked bilateral hydroureteronephrosis, bladder distension, and renal insufficiency. Imaging shows massive reflux. The hydroureteronephrosis decreases with catheter drainage. This indirectly indicates that UPJ and UVJ obstruction are not present. Bladder decompression with a vesicostomy will decrease the risk of ascending infection and optimize renal function. Since the dilation decreased with bladder drainage, upper tract diversion is not indicated. He is at risk for UTI and CIC will introduce bacteria into the urinary tract. At five days of age, a MAG-3 renal scan is of limited utility with regards to assessment of obstruction in the presence of an elevated creatinine. The cystogram findings and history are not suspicious for a neurogenic bladder so a spinal ultrasound is not indicated. Khoury AE, Bagli DJ: Vesicoureteral reflux, Wein, AJ, Kavoussi LR, Novick AC, Partin AW, Peters CA (eds): CAMPBELL-WALSH UROLOGY, ed 10. Philadelphia, Elsevier Saunders, 2012, vol 4, chap 122, pp 3283-3285. Question #84 ANSWER=D ______________________________________________________________________________ This infant has a solitary kidney and high grade obstruction as evidenced by the absence of retrograde refluxed flow into the renal pelvis on the cystogram. With the elevated creatinine indicating reduced GFR, the MAG-3 scan is of limited utility (solitary kidney relative function is indeterminable) and is not required to confirm probable obstruction. Both percutaneous nephrostomy and ureteral stent placement

Page 41: Live Course Handout.cfm Second Part

are temporizing measures. In addition, retrograde ureteral stent placement in a six-day-old boy can be technically difficult. Both procedures may cause temporary edema and inflammation that complicate subsequent reconstruction. The imaging shows a normal caliber ureter, so a pyeloplasty is technically possible and definitive. A nephrostomy tube can be left after pyeloplasty to optimize recovery of renal function. The primary pathology is at the UPJ instead of the UVJ, so ureteral reimplantation is not indicated. Carr MC, Casale P: Anomalies and surgery of the ureter in children, Wein, AJ, Kavoussi LR, Novick AC, Partin AW, Peters CA (eds): CAMPBELL-WALSH UROLOGY, ed 10. Philadelphia, Elsevier Saunders, 2012, vol 4, chap 120, p 3212. Question #85 ANSWER=B ______________________________________________________________________________ The Chi-square test is intended to determine if an observed distribution is due to chance. It measures how well the observed distribution of data fits with the distribution expected if the variables are independent. The T-test compares the means from two independent or different groups and works best with a single dependent variable. The assumption of the T-test is that the dependent variable should be normally distributed in both populations. The Wilcoxson rank sum test is a non-parametric statistical hypothesis test used in place of the T-test when comparing two related samples. It is preferentially used to compare two related samples, when the population cannot be assumed to be normally distributed. The one-way analysis of variance (ANOVA) is used to determine whether there are any significant differences between the means of three or more independent groups. Linear regression is a common statistical analysis technique used to determine the extent to which there is a linear relationship between a dependent variable and one or more independent variables. https://www.auanet.org/login/?refer=/university/core_topic.cfm?coreid=122 Question #86 ANSWER=E ______________________________________________________________________________ Renal calcifications found on radiographic evaluations are listed as either cortical or medullary calcifications. The image noted is classic for medullary calcifications. The differential diagnosis for medullary calcifications are: distal renal tubular acidosis, hyperparathyroidism, hyperoxaluria, medullary sponge kidney, renal papillary necrosis, and tuberculosis. The image below would be most consistent with medullary sponge kidney or distal renal tubular acidosis. The etiologies for cortical calcifications are: Alport's syndrome, acute tubular necrosis (secondary to shock), chronic glomerulonephritis, renal transplant rejection, and calcifications secondary to sickle cell anemia or trait. Pearle MS, Lotan Y: Urinary lithiasis: Etiology, epidemiology, and pathogenesis, Wein, AJ, Kavoussi LR, Novick AC, Partin AW, Peters CA (eds): CAMPBELL-WALSH UROLOGY, ed 10. Philadelphia, Elsevier Saunders, 2012, vol 2, chap 45, pp 1275-1277.

Page 42: Live Course Handout.cfm Second Part

Question #87 ANSWER=E ______________________________________________________________________________ Increasing fluid intake to reach a minimum urine output of 2000 to 2400 ml per day is the most important measure to decrease the risk of stone recurrence in the post-pubertal patient. In a patient with the findings of isolated hypercalciuria and a calcium oxalate stone, starting thiazide diuretics and dietary alterations are the next steps to reduce stone recurrence rates. In general, dietary recommendations for patients with isolated hypercalciuria are a lower sodium diet, normal calcium intake, and limit intake of non-dairy animal protein. If the patient has low normal levels of citrate, an increase in fruits, lemonade, orange juice, or sodas using a citric acid base (clear sodas) may also be recommended. Please note that patients should avoid consuming primarily phosphoric acid-based sodas (cola), as these have been demonstrated to slightly increase the risk of stone formation. Potassium citrate and allopurinol have also been shown to be effective in reducing the risk of calcium stones in select patients. The clinical effectiveness of citrate supplementation to reduce stone formation appears to be limited to patients who are hypercalciuric with concurrent low urinary citrate. Allopurinol use in patents with calcium oxalate stones is limited to individuals noted to have concurrent findings of hyperuricosuria and/or hyperuricemic patients. Pearle MS, Goldfarb DS, Assimos DG, et al: Medical management of kidney stones. AUA GUIDELINES. American Urological Association Education and Research, Inc, 2014.http://www.auanet.org/education/guidelines/management-kidney-stones.cfm Question #88 ANSWER=C ______________________________________________________________________________ Lesch-Nyhan syndrome is an inherited disorder associated with a deficiency in the enzyme hypoxanthine-guanine phosphoribosyl-transferase. Failure of this enzyme to function will ultimately result in increased levels of hypoxanthine and xanthine; these are then converted to uric acid by the enzyme xanthine oxidase. Patients with untreated Lesch-Nyhan syndrome will present with uric acid stones. It is noteworthy that once a patient is placed on treatment with allopurinol, the most common type of urinary stone is altered. Allopurinol will block the enzyme xanthine oxidase and stop uric acid build up. This results in a significant increase in xanthine and the development of xanthine stones. Calcium oxalate, ammonium acid urate, and cystine stones do not occur in conjunction with this inherited disorder or its treatment. Pearle MS, Lotan Y: Urinary lithiasis: Etiology, epidemiology, and pathogenesis, Wein, AJ, Kavoussi LR, Novick AC, Partin AW, Peters CA (eds): CAMPBELL-WALSH UROLOGY, ed 10. Philadelphia, Elsevier Saunders, 2012, vol 2, chap 45, p 1282.Torres RJ, Prior C, Puig JG: Efficacy and safety of allopurinol in patients with the Lesch-Nyhan syndrome. NUCLEOSIDES NUCLEOTIDES NUCLEIC ACIDS 2006:25(9-11):1077-1082. Question #89 ANSWER=C ______________________________________________________________________________

Page 43: Live Course Handout.cfm Second Part

Cystinuria is an autosomal recessive defect associated with the inability of the dibasic amino acids cysteine, ornithine, lysine, and arginine (COLA) to be actively reabsorbed from the proximal tubules. Excessive secretion of ornithine, lysine, and arginine are clinically irrelevant, since these amino acids are completely soluble in the urine at physiologic pH. However, cystine, a disulfide-linked homodimer of cysteine, has a urine solubility index that is highly pH dependent, with urinary acidification associated with precipitation of cystine crystals and subsequently stone formation. In patients with cystinuria, the first line of therapy is to increase fluid intake to push urine output to > 2 to 2.4 liters per day. Urinary alkalinization with potassium citrate will be pursued simultaneously. Oral potassium citrate therapy should be titratated to maintain the urine pH level between 7.5 and 8.0. Increasing urine pH > 8.0 is associated with urinary calcium precipitation and increased stone growth, with precipitation of calcium phosphate crystals around a cystine nidus. Similarly, use of sodium citrate is not recommended for therapy. Excess oral sodium intake can result in increased urinary calcium secretion. The increase in urinary calcium results in its crystallization around a cystine nidus, and increased stone growth and or new stone formation. Alpha-mercaptopropionylglycine (alpha-MPG or Thiola), d-penicillamine, and captopril share similar chemical properties regarding inhibition of cystine crystals. Specifically, a disulfide bond exchange occurs to form a soluble cysteine compound. D-Penicillamine is an effective yet potentially toxic drug; bone marrow suppression, anorexia, vomiting, and diarrhea occur in approximately 25-30% of the patients treated with this medication. Captopril may be a useful drug for the treatment of cystinuria; however, the dosage needed to achieve clinical effectiveness is associated with symptomatic orthostatic hypotension in almost all patients. A multicenter study found that the effectiveness of alpha-MPG in reducing cystine concentration was equal to that of d-penicillamine, with alpha-MPG having significantly fewer side effects. Alpha-MPG is therefore recommended as the second line of therapy if fluid intake and urinary alkalinization fail. Either D-penicillamine or captopril may be tried if combination therapy with increased fluid intake, alkalinization, and alpha-MPG fail. Ferrandino MN, Pietrow PK, Preminger GM : Evaluation and medical management of urinary lithiasis, Wein, AJ, Kavoussi LR, Novick AC, Partin AW, Peters CA (eds): CAMPBELL-WALSH UROLOGY, ed 10. Philadelphia, Elsevier Saunders, 2012, vol 2, chap 46, pp 1317-1318.Pearle MS, Lotan Y: Urinary lithiasis: Etiology, epidemiology, and pathogenesis, Wein, AJ, Kavoussi LR, Novick AC, Partin AW, Peters CA (eds): CAMPBELL-WALSH UROLOGY, ed 10. Philadelphia, Elsevier Saunders, 2012, vol 2, chap 45, pp 1277-1280.Pak CYC, Fuller C, Sakhaee K, et al: Management of cystine nephrolithiasis with alpha-mercaptopropionylglycine. J UROL 1986;136:1003. Perazella MA, Buller GK: Successful treatment of cystinuria with captopril. AM J KID DIS 2993;21:504. Streem SB, Hall P: Effect of captopril on urinary cystine excretion in homozygous cystinuria. J UROL 1989;142:1522. Question #90 ANSWER=B ______________________________________________________________________________ A patient with an imperforate anus is at risk for six associated genitourinary problems: 1) renal agenesis/dysplasia, 2) VUR, 3) neurogenic bladder due to partial sacral agenesis and/or a tethered spinal cord, 4) rectourethral fistula, 5) increased incidence of cryptorchidism, and 6) increased risk of erectile dysfunction at puberty. In an infant presenting with a febrile UTI and an elevation in serum creatinine, there is a great concern for a poorly emptying neurogenic bladder that is associated with a high level of bacterial colonization secondary to the rectourethral fistula, thereby leading to the febrile UTI. The elevation in creatinine could be due to urine draining via the fistula into the colon, resulting in elevation

Page 44: Live Course Handout.cfm Second Part

of creatinine via an absorptive phenomena. This is usually associated with concurrent metabolic acidosis, or possibly secondary to concurrent renal dysplasia and the development of renal failure. Management of this patient should be by vesicostomy, proximal diversion of the urine away from the rectourethral fistula. CIC is frequently problematic due to the rectourethral fistula, and can be associated with significant urethral trauma. Antireflux surgery in this age group and early repair of the high rectourethral fistula would be inappropriate, and would fail to treat the possible concurrent neurogenic bladder. Although bilateral cutaneous pyelostomies could be performed, this would leave a dry bladder. This would result in temporary loss of bladder volume, and make reconstructive surgery more difficult in the future. MacLellan DL, Bauer SB: Neuropathic dysfunction of the lower urinary tract, Wein, AJ, Kavoussi LR, Novick AC, Partin AW, Peters CA (eds): CAMPBELL-WALSH UROLOGY, ed 10. Philadelphia, Elsevier Saunders, 2012, vol 4, chap 128, pp 3450-3452. Question #91 ANSWER=C ______________________________________________________________________________ Acetohydroxamic acid is a urease inhibitor that has been shown to significantly reduce the growth of struvite residual stone fragments left behind following treatment. Its use is, however, associated with significant side effects, notably the development of deep vein thrombosis in up to 15% of patients. Up to 50% of patients discontinue the medication due to one of a multitude of complaints, including GI distress, headaches, rash, abdominal pain, and alopecia. Due to the significant side effects that exist with this drug, its use is limited. Ferrandino MN, Pietrow PK, Preminger GM : Evaluation and medical management of urinary lithiasis, Wein, AJ, Kavoussi LR, Novick AC, Partin AW, Peters CA (eds): CAMPBELL-WALSH UROLOGY, ed 10. Philadelphia, Elsevier Saunders, 2012, vol 2, chap 46, p 1319. Question #92 ANSWER=D ______________________________________________________________________________ This patient has a small volume, high pressure bladder associated with low grade VUR and incontinence. CIC with augmentation cystoplasty will achieve continence with the improvement in detrusor pressures associated with a high probability of resolving the low grade reflux. The addition of a bilateral ureteral reimplantation for low grade reflux in this patient only increases her surgical risk. If reflux resolution fails and the patient is symptomatic, use of a subtrigonal bulking agent could be considered at that time. The patient's detrusor LPP are elevated and there is no indication that increasing outlet resistance by either an artificial urinary sphincter, rectus facial sling, or injection of bladder neck bulking agents is necessary to gain urinary continence. Indeed, increasing outlet resistance without improving bladder compliance could result in progressive upper tract deterioration due to worsening of the detrusor LPP.

Page 45: Live Course Handout.cfm Second Part

Adams MC, Joseph DB: Urinary tract reconstruction in children, Wein, AJ, Kavoussi LR, Novick AC, Partin AW, Peters CA (eds): CAMPBELL-WALSH UROLOGY, ed 10. Philadelphia, Elsevier Saunders, 2012, vol 4, chap 129, pp 3464-3472. Question #93 ANSWER=A ______________________________________________________________________________ Greater than 75% of patients with a spinal cord injury above T-10 will develop an overactive bladder with detrusor sphincter dyssynergy. Please note the absence of the bulbospongiosus reflex in this patient. This reflex involves the S2-S4 nerve root, and is the first reflex to return with resolution of spinal cord shock. The test involves monitoring anal sphincter contraction in response to squeezing the glans penis, clitoris, or tugging on an indwelling urethral catheter. The absence of the reflex indicates either persistent spinal cord shock or a concurrent injury to the conus medullaris. Due to the high probability of persistent spinal cord shock, this patient should continue on CIC. Classically, spinal cord shock begins to wear off two months after injury, but the full extent of muscle spasms and detrusor overactivity may take as long as 12 to 24 months to develop. Surgical intervention for patients with an acute spinal cord injury should not be entertained until there is resolution of spinal cord shock. Capsaicin or antimuscarinics may be used to treat bladder overactivity, currently not noted in this patient. Sacral neuromodulation is not indicated in individuals with traumatic spinal cord injuries. MacLellan DL, Bauer SB: Neuropathic dysfunction of the lower urinary tract, Wein, AJ, Kavoussi LR, Novick AC, Partin AW, Peters CA (eds): CAMPBELL-WALSH UROLOGY, ed 10. Philadelphia, Elsevier Saunders, 2012, vol 4, chap 128, pp 3454-3456. Question #94 ANSWER=C ______________________________________________________________________________ Three to five percent of patients with Crohn's disease will initially present with urinary tract symptoms, 90% of whom will have a initially positive urine culture. Patients presenting with GU symptoms will almost invariably have persistent irritative voiding symptoms with an abnormal urinalysis after clearing their urine of bacteriuria. Classically, ultrasound or CT scan evaluation will reveal a thickened posterior bladder wall, co-existing with adjacent loops of small bowel wall having a thickness of greater than or equal to 4 mm thickness. These combined radiographic findings are 75% sensitive and 95% specific for the diagnosis of Crohn's disease. The patient described above has the classic symptoms and findings of a patient with Crohn's disease. Initial treatment should be with a GI consultation. Cystoscopy with bladder biopsy should be considered in individuals with concurrent ureteral obstruction or pneumaturia. In patients with Crohn's disease, long-term follow-up reveals 40% will eventually have involvement of their urinary tract, with 20% noting the presence of irritative GU symptoms, 15-20% developing renal calculi, and five percent of patients will develop ureteral obstruction or enteric vesical fistula. If ureteral obstruction does occur, it invariably involves the right ureter due to pelvic inflammation and/or a pelvic abscess. These patients should be treated with a ureteral stent and medications as directed by the gastroenterologist. If the ureteral obstruction remains after six months, bowel resection and/or ureteral reimplantation should be considered. Patients who develop enteric-vesical fistulas require bowel resection and closure of the vesical fistula.

Page 46: Live Course Handout.cfm Second Part

Ben-Ami H, Ginsesen Y, Belar DM, et al: Diagnosis and treatment of urinary tract manifestations of Crohn's Disease. CAN J GASTROENTEROLOGY 2002:16:225-231.Calabrese E: Bowel ultrasound for the assessment of Crohn's Disease. GASTROENTEROL HEPATOL 2011:7:107?109. Question #95 ANSWER=D ______________________________________________________________________________ Eosinophilic cystitis (EC) is a rare inflammatory bladder condition that has a strong female predominance (>85%). Twenty percent of the patients present in childhood. EC has been shown to have a higher than normal incidence in patients with a history of asthma, celiac disease, and systemic lupus erythematous. The etiological causes of EC have been shown to be multifactorial. They include seasonal and food allergies; bacterial, parasitic and viral infections; bladder tumors; intravesical thiotepa; and mitomycin. EC is hypothesized to be stimulated and mediated by antigen-antibody reaction to an allergic stimulus. The urologic symptoms consist of marked urinary frequency, urgency, hematuria, dysuria, and suprapubic discomfort. Cystoscopy and biopsy are the gold standard for diagnosis. Cystoscopy will reveal findings of acute inflammation and, on occasion, will be associated with either small papillary or large polypoid masses that can resemble either a urothelial malignancy or rhabdomyosarcoma of the bladder. It should be noted that EC has biphasic appearance on biopsy. In the initial acute phase, it will be associated with an intense eosinophilic infiltrate associated with acute inflammatory polypoid changes. As the process can become chronic, biopsies in the chronic phase will reveal chronic fibrosis and/or chronic inflammation with little to no eosinophilic infiltrate. Laboratory evidence supporting the diagnosis includes proteinuria and microscopic hematuria. Only rarely will patients be found to have coexisting peripheral eosinophilia; however, when found, this is highly correlated to biopsy findings. Treatment should include removal of the patient from any possible allergen, antihistamines, and non-steroidal anti-inflammatories. Greater than 95% of patients will respond to this therapy. Although antimuscarinics such as oxybutynin can be added, this class of medications has not been found to be as helpful symptomatically as antihistamines and NSAIDS. If symptoms persist, consideration for steroids and fulguration or transurethral resection of any visible bladder lesions should be considered. In spite of successful initial therapy, EC may recur and/or become chronic leading to an end stage bladder. Long-term follow-up is considered by many to be mandatory. Teegavarapu PS, Sahai A, Chandra A, et al: Eosinophilic cystitis and its management. INT J CLIN PRACT 2005;59:356-360. Popescu, O, Landas, SK, Haas, GP: The spectrum of eosinophilic cystitis in males: Case series and literature review. ARCH PATHOL LAB MED 2009;133:289?294. Question #96 ANSWER=B ______________________________________________________________________________ The majority of children with cerebral palsy are able to develop total urinary control. It is the minority of children with cerebral palsy who have urinary incontinence and inability to potty train. Urodynamic studies in this select patient population demonstrate detrusor overactivity in 80%, normal activity in 12%, detrusor overactivity with detrusor-sphincter dyssynergy in five percent, and poor bladder compliance with overflow incontinence in three percent. On rare occasions, an underactive bladder with

Page 47: Live Course Handout.cfm Second Part

overflow incontinence may also be seen. This latter finding is almost invariably associated with a history of low APGAR scores. In children with cerebral palsy with problematic incontinence and/or recurrent UTIs, a VCUG will show a trabeculated bladder in 58% and VUR in nine percent. There is not an increased incidence of primary bladder neck dysfunction in this group. The ability to gain urinary continence in patients with cerebral palsy is based on mental acuity, patient mobility, and urodynamic findings. Continence can be obtained in the majority with antimuscarinics, timed voiding, and maintenance of a voiding diary, provided the patient has adequate mental acuity. Please note that although cerebral palsy does not have advancing neurologic dysfunction, striated muscle spasm continues to worsen with age. Post-pubertal patients (especially males) may develop significant problems with urinary retention, recurrent epididymitis, and UTIs due to tonic external urinary sphincter spasm. The development of tonic external urinary sphincter spasm will result in urinary reflux into the vas deferens with voiding, recurrent epididymitis, elevated post-void urines, and possible upper tract deterioration. Classically, these symptoms develop in eight to ten percent of patients with cerebral palsy with onset of complaints in the second to third decade of life. MacLellan DL, Bauer SB: Neuropathic dysfunction of the lower urinary tract, Wein, AJ, Kavoussi LR, Novick AC, Partin AW, Peters CA (eds): CAMPBELL-WALSH UROLOGY, ed 10. Philadelphia, Elsevier Saunders, 2012, vol 4, chap 128, pp 3452-3454. Question #97 ANSWER=B ______________________________________________________________________________ Removal of the distal 15-20 cm of the ileum can possibly result in Vitamin B12 deficiency with the possible consequences of megaloblastic anemia (not iron deficiency anemia), neuropathy, and elevated levels of serum homocysteine causing an increased risk of carotid and coronary atherosclerosis. Low B12 levels may be associated with any one of the aforementioned sequela, or a combination of these defects. Although the megaloblastic anemia is reversible with treatment, the neuropathy and the atherosclerotic defects are either only partially or totally irreversible. Routine yearly follow-up of patients for low serum B12 levels is recommended in all patients where the terminal ileum was used for reconstruction. It is noteworthy that depletion of total body B12 may take greater than five years to develop and routinely increases with age, with three percent of the normal population having B12 deficiency in their fifties. Five percent are deficient in their sixties. The metabolic complication that may occur with ileocystoplasty is hyperchloremic metabolic acidosis. Hypochloremic alkalosis is seen with gastric cystoplasty. Patients with ileal resection may also have trouble absorbing fat-soluble Vitamins (A, D, E, and K). Vitamin K is needed to make prothrombin and other clotting factors. Thus, a prothrombin level should be measured (to ensure it is not elevated) if additional major reconstructive surgery is contemplated. Platelet dysfunction does not occur as a consequence of using the terminal ileum for reconstructive purposes. Adams MC, Joseph DB: Urinary tract reconstruction in children, Wein, AJ, Kavoussi LR, Novick AC, Partin AW, Peters CA (eds): CAMPBELL-WALSH UROLOGY, ed 10. Philadelphia, Elsevier Saunders, 2012, vol 4, chap 129, pp 3480-3482. Question #98 ANSWER=C

Page 48: Live Course Handout.cfm Second Part

______________________________________________________________________________ The positive predictive value of a test determines how likely it is that the disorder is present or absent. The positive predictive value represents the proportion of positive results that are true positives or the likelihood that a person with a positive result has the disorder, in this case anorchia. Positive predictive value equals the number of those who test positive and have anorchia (true positive) over the total number of those who test positive (true positive + false positive). In this case PPV = 30/40=.75, which means that if a patient’s test result is positive for anorchia, he has a 75% chance of having anorchia. Glaser AN. High-yield biostatics. 3rd edition. Lippincott Williams and Wilkins. 2005, pp 82. Question #99 ANSWER=A ______________________________________________________________________________ The ultrasound demonstrates significant hydronephrosis bilaterally. In light of the preoperative grade 4 VUR, this finding is not necessarily worrisome. This finding could represent persistent reflux, obstruction at the site of the reimplantations, bladder dysfunction, or persistent dilation of the upper tracts after successful surgery. In light of the post-op febrile UTIs, however, there would be concern for a underlying abnormality or complication from the surgery. The next steps should be for a VCUG and a MAG-3 diuretic wash-out renogram, as was performed in this case. The results of the MAG-3 renogram reveal that bladder is quite full, and, therefore, may represent a false positive washout curve, due to an underlying bladder dysfunction as a cause for the recurrent UTI and persistent hydronephrosis. To better assess the clinical situation, the MAG-3 study should be repeated with an empty bladder with a catheter in-situ and placed to dependent drainage. Provided the VCUG was an adequate study with voiding, a repeat VCUG would not be indicated. A DMSA scan would add no significant information regarding concerns about possible ureterovesical junction obstruction or primary bladder dysfunction. A uroflow and EMG may demonstrate an abnormal voiding pattern; however, it would not necessarily help in defining the cause of the hydronephrosis. The order of testing should be a MAG-3 diuretic scan with an indwelling catheter, if this reveals no evidence of obstruction, it should be followed by a urodynamic study to rule-out abnormal bladder dynamics. A timed voiding schedule alone is not likely to confirm the presence of abnormal bladder dynamics, but may be used as part of the treatment if the diagnosis of a primary bladder dysfunction is confirmed. Khoury AE, Bagli DJ: Vesicoureteral reflux, Wein, AJ, Kavoussi LR, Novick AC, Partin AW, Peters CA (eds): CAMPBELL-WALSH UROLOGY, ed 10. Philadelphia, Elsevier Saunders, 2012, vol 4, chap 122, p 3302. Question #100 ANSWER=E ______________________________________________________________________________ The VCUG demonstrates a urethral polyp and VUR. Fibroepithelial urethral polyps can cause intermittent bladder outlet obstruction. Uroflow and EMG would not be helpful, and may be misleading due to voluntary urethral sphincter contraction during voiding. CT scan would not add useful information in establishing the diagnosis. The image is not consistent with PUV; therefore, transurethral

Page 49: Live Course Handout.cfm Second Part

incision would not be appropriate. Therefore, TUR is the correct answer. This may be technically challenging depending on the size of the polyp, but it is usually attached to a narrow stalk. Casale AJ: Posterior urethral valves, Wein, AJ, Kavoussi LR, Novick AC, Partin AW, Peters CA (eds): CAMPBELL-WALSH UROLOGY, ed 10. Philadelphia, Elsevier Saunders, 2012, vol 4, chap 126, p 3407.